Exam 1 OB and Peds

Infant: 1 to 12 monthsToddler: 1 to 3 yearsPreschooler: 3 to 6 yearsSchool age: 6 to 12 yearsAdolescent: 12 to 20 years

Erikson and Piaget Theories for each age group

Infant (1-12 months):-Erikson: trust vs. mistrust -Piaget: sensorimotor Toddler (1-3 years): -Erikson: autonomy vs. shame/doubt -Piaget: sensorimotor and preoperational; early egocentrismPreschooler (3-6 years): -Erikson: initiative vs. guilt -Piaget: preoperational cause and effect, magical thinkingSchool Age (6-12 years): -Erikson: industry vs. inferiority -Piaget: concrete operationalAdolescent (12-18 years): -Erikson: identity vs role confusion -Piaget: formal operational, abstract thinking*said to be familiar with these because they are on test

According to Piaget, at which age does a child learn conservation?

Concrete operational stage (7-11) school aged child

Growth and Development

-growth refers to an increase in physical size.-development is a sequential process by which infants and children gain various skills and functions. Heredity influences growth and development by determining child's potential (what we want them to reach). Environment contributes to this achievement to a degree.

Physical growth in children happens in which pattern/direction?

Cephalocaudal (head to feet) Proximodistal (simple to complex)

Growth and Development of the infant

Physical Growth: grow very rapidly in the first 12 mo. Weight, length, and head circumference are closely monitored.Body Systems: most are immature at birthGross and fine motor:-gross motor skills develop in cephalocaudal fashion (head to toe)-fine motor skills develop in proximodistal fashion (from center to periphery --> bat hand before developing ability to grasp with fingertips)Learning: psychosocial and cognitive, language and communication, social/emotional domains.Measurements: -average weight: 3.4 kg or 7.5 lbs -average length: 20 inches -average head circumference: 13.5 inches (35 cm)*infants may lose 5-10% of body weight over the first week of life. Most will regain this by 7-10 days life.

Physical growth guidelines for infants

Weight: -by 6 months double birth weight -by 12 months triples birth weight*average infant birth weight 7.5 lbs *average 1 year old weighs23 lbs Length: -grow more quickly in first 6 mo. than seconds 6 mo.-by 12 mo. length increases by 50% Head Circumference: -increases rapidly in first 6 mo. -increases about 10 cm from birth to 12 mo.*fontanelles and cranial sutures not fused so the brain can grow --> check every visit until 3 years old

Assessing growth and development of premature infants

use infants adjusted age to determine expected outcomes (subtract the number of weeks the infant was premature from the infant's chronological age). Plot growth parameters and assess developmental milestones based on adjusted age.

Physiologic changes as infant grows

Neurologic system: increase in head circumference indicates brain growth. Anterior fontanels remains open until 12-18 months. Posterior fontanelle can be closed at both or closes soon after. Primitive reflexes present at birth and most disappear over the first few months and adapt into protective reflexes.Respiratory System: RR slows from 30-60 breaths per minute as newborn to 20-30 breaths per minute in 12 mo. old. Newborn will have irregular breathing with periodic pauses, this becomes regular and rhythmic with maturity. Respiratory system does not reach adult levels of maturity until 7 years of age.Cardiovascular System: Heart doubles in size over the first year of life. Pulse rate decreases. from 120-140 in newborn to 100 in 12 month old. BP increase from 60/40 of newborn to 100/50 in 12 month old. Thermoregulation becomes more effective over 1st year of life.Gastrointestinal System: first primary tooth begins to erupt around 6 months; occasionally infant is born with one or more teeth (natal teeth) which may be ass.w/ other birth abnormalities. Small amounts of saliva for first 3 months of life. Small stomach capacity at birth (1/2 to 1 ounce). Breastfed stools are looser in texture and appear seedy. Meconium (newborns first stool) are dark green and tarry. Genitourinary system: infant total-body water is greater percentage weight than an adult. Infant is more susceptible to dehydration. Infants frequently urinate. Urine has low specific gravity (low concentration).Integumentary System: fine downy hair (lanugo). Newborn acrocyanosis (blueness of hands and feet) decreases over the first few days of life. Central cyanosis is NOT normal.Hematopoietic System: iron stores from mother; stores will be depleted by 6-9 months if iron supplementation doesn't occur. Immunologic System: Newborns receive IgG (longer immunity) from mothers which helps with immunity during first 3-6 months of life.KNOW FOR TEST WARNING!!:-lack of increased head circumference at sequential measurements -premature anterior fontanel closure -persistent primitive reflexes or return of primitive reflexes. -central cyanosis (cyanosis in the head, mouth, or torso)

Extrustion reflex definition and when does it go away?

Sticking tongue out- 4 months

Moro reflex and when does it go away?

Startle- 4 months

Rooting reflex definition and when does it go away?

Turning head towards stimulus when touched on the cheek -3 to 4 months

Babinskis reflex and when does it go away?

Stroking bottom of foot and toes fan out -24 months

How is the respiratory system of an infant different from an adult?

-Narrower nasal passages -Narrower, shorter bronchi -Larger tongue -Fewer alveoli -Diaphragmatic breathing

The average pulse rate decreases from ________ in the newborn to _____ in the 1 year old

120-140 in the newborn to 100 in the 1 year old

What are some warning signs indicating problems with sensory development in an infant?

-young infant doesn't respond to loud noises -child does not focus on near object -infant crosses eyes most of the time at 6 months

When should an infant: -start to make sounds or babble -laugh or squeal -use single words w/ meaning (mama, dada)

-sounds or babble (4 months) -laugh or squeal (6 months) -single words (12 months)

Posterior fontanel closes at ____ moths; Anterior fontanel closes at _____ moths

posterior 2-5 months; anterior 18-24 months

Psychosocial (Erikson) and Cognitive Development (Piaget) in infants

Trust vs. mistrust -caregivers respond to infant's basic needs -infant gradually learns to tolerate small amounts of frustration trusting that gratification will eventually be provided. Sensorimotor: -infant uses senses and motor skills to learn about the world.-familiarizing -object permanence -purposeful movements -symbols and events begin to become associated.*This is when you learn someone will care for you.

Infant Gross Motor Skill Development

Large muscle movements develop in a cephalocaudal fashion (head to tail).1 month-> lifts and turns head to side in prone position; head lag when pull to sit; rounded back in sitting.2 months-> raises head and chest, holds position; head control is improving.3 months-> raises head to 45 degrees in prone; slight head lag in pull-to-sit. 4 months-> lifts head and looks around; rolls from prone to supine; head leads body when pulled to sit. 5 months-> rolls from supine to prone and back again; sits with back upright when supported. 6 months-> tripod sits 7 months-> sits alone with some use of hands for support. 8 months-> sits unsupported.9 months-> crawls, abdomen off floor.10 months-> pulls to stand; cruises12 months-> sits from standing position; walks independently WARNING (KNOW FOR TEST) -persistent head lag after 4 months -unable to sit unsupported at 9 months

Infant Fine motor Skill Development

Hand to finger use. Develop in proximodisal fashion (center to periphery).1 month-> fists mostly clenched; involuntary hand movements. 3 months-> holds hand in front of face, hands open.4 months-> bats at objects. 5 months-> grasps rattle. 6 months-> releases object in hand to take another. 7 months-> transfers objects from one hand to the other. 8 months-> gross pincer grasps (rakes). 9 months-> bangs objects together. 10 months-> fine pincer grasp; puts objects into container and takes them out. 11 months-> offers objects to others and releases them. 12 months-> feeds self with cup and spoon; makes simple mark on paper; pokes with index finger.WARNING (KNOW FOR TEST) -lack of pincer grasp at 12 months

Sensory Development in Infants

Sight: -newborn is nearsighted (8 to 15 inches) -prefer human faces and high contrast objects-newborn's eyes may occasionally cross -can recognize people they know best by 1 month-binocular vision by 6 weeks -full color vision by 7 monthsHearing -acute as an adult at birth Smell and Taste: -smell develops rapidly -7 day old can distinguish mother's breast milk -prefers sweets Touch:-prefers soft sensations-holding, stroking, rocking, or cuddling calms infant.WARNING (KNOW FOR TEST) -persistent eye crossing after 5 months -no reaction to sudden, loud noise any time after birth

Communication and Language in Infants

Very important for caregiver to talk to the infant to learn communication skills. Regression in language development occurs briefly when a child is focusing energy on other skills, like crawling or walking. As long as hearing is normal, language acquisition should continue to progress.Infants in bilingual families may "language mix" which is normal language progression, but does make it harder for Dr or NP to determine delays in communication skills. Newborn-> crying communicates basic needs. 1-3 months-> coos, vocalizations. 4-5 months-> simple vowel sounds, laughs, raspberries. 6 months-> squealing and yelling. 7-10 months-> babbling, responds to simple commands9-12 months-> knows meaning of mama and dada; starts to imitate other speech sounds. 12 month old-> uses 2-3 recognizable words with meaning.WARNING (KNOW FOR TEST) -no coos or vocalizations by 4 months -no laugh or squeal by 6 months-no babbling by 8 months -no single words with meaning at 12 months (mama, dada) -loss of previously acquired milestones.

Social and Emotional Development in Infants

Temperament ranges from low or moderately active, regular and predictable to highly active, more intense, and less adaptable. Meet the kid where they're at developmentally, not where they should be. Stranger Anxiety: (8-9 months)-> clingy and whiny when approached by less known people. -infant recognizing self as separate from others.-individuals should approach infant calmly and slowly.Separation Anxiety: -9 month-> distressed when parent leaves.-lessens when infant is older and has sufficient memory and cognition to realize parent will come back.Temperament: -individual's nature -broad range -affects the way infant responds to the environment. Cultural Influences: -can have impact on growth and development-certain ethnic groups tend to be shorter -health beliefs influenced by religious or spiritual background.-sleeping habits

Promoting healthy growth and development in infants

Toys should evolve to match developmental tasks and milestones. Some ethnic groups tend to be lactose intolerant (African Americans, Native Americans, and Asians); therefore, alternative sources of calcium must be offered. Play: -the work of children -gross and fine motor skills practiced through play-allow infants to explore their environments, practice new skills, and solve problems. -prefers parents and toys-solitary playEarly Learning: -reading aloud is critical to developing neural networks-over time, reading leads to acquisitions of language skills. Safety: -as more mobile, risk of injury increases. -car safety -crib and home safety -water safety Nutrition: -cultural dietary practices -ethnic considerations -calorie needs change with growth-breastfeed exclusively for first 6 months-TEETH and GUMS maintain oral hygieneSleep and Rest: -sleep needs change along infancy -newborn-> 21 hours a day -one year old-> 10-12 hours at night w/ naps

Common developmental concerns in infants

Colic: Inconsolable crying 3 or more hours per day, 3 to 7 days a week w/o physical cause. -resolves by 3 moths -keeping baby safe-> tell them it's okay to put the baby in a safe crib and walk away.Thumb sucking/Security items: -thumb sucking is healthy self-comforting activity.-never try to break a habit during a stressful time. -early school age is when it becomes a worry. -no studies have proven that thumb sucking leads to braces unless beyond early school age period.Spitting Up: -regurgitating small amounts of stomach contents. -occurs in all infants, and a significant amount of normal infants spit up excessively.-Overfed babies who feed based on a parent designed schedule and those who get burped poorly are more likely to spit up.-For some infants, the amount and frequency of spitting up are significant and they should be seen by dr.Teething: -pain is from inflammation -fever, vomiting, and diarrhea are not signs of teething but rather an illness.

Growth and Development in the toddler (1 to 3 years)

Physical Growth: -height and weight increase steadily but at slower velocity than infancy.-gains in height and weight occur in spurts Body Systems: Changes not as pronounced as infancy but organ systems continue to grow and mature.Gross and Fine motor: -gross motor skills are mastered and used repeatedly-fine motor skills are improved and perfectedLearning: -psychosocial and cognitive -language and communication -social/emotional domainsPhysical Growth Guidelines: (Weight)-> gains 3 to 5 lbs per year (Length)-> grows 3 inches per year; by 2 years half of adult height is reached.(Head Circumference)-> increases about 1 inch between 1 to 2 years; increases 1/2 inch per year until 5 years old

Physiologic changes as toddler grows

Neurologic System: -head circumference reaches about 90% of adult size by 2 years. -improved coordination and deliberate sphincter control.-protective reflexes emerge -rapid increase in language skills Respiratory System:-Alveoli continue to increase in number (at age 7 they have close to the same amount as adults.-Airways continue to grow but are still small compared to an adult. -Tonsils and adenoids are huge proportionally.Cardiovascular System: -heart rate decreases-blood pressure increasesGastrointestinal System: -stomach size increases allowing toddler to consume 3 meals a day-stooling decreases to one or more a dayGenitourinary: -bladder and kidney functions reach adult levels by 16 to 24 months of age.-bladder capacity increases which allows toddler to retain urine for increased periods of time.Musculoskeletal System: -bones increase in length and become stronger. -weak abdominal musculature (pot-bellied). -by 3 years abdomen is flatter in appearance.

Average toddler gains an average of ____ lbs per year

3-5 lbs

Toddler Motor Skill Development

Large muscle groups are strengthened; eye hand coordination improves. Toddler gait: legs wide apart, toes pointed forward, sways from side to side.By 3 years they should walk in heel to toe fashion. 12-15 months: -gross-> walks independently -fine-> feeds self finger foods; uses index finger to point 18 months: -gross-> climbs stairs with help; pulls toys while walking -fine-> Masters reaching, grasping, and releasing; stacks blocks, puts things in slots. Turns book pages. Removes socks and shoes. Stacks 4 cubes .24 months: -gross-> runs, kicks balls, can stand to tiptoe. Carries several toys or a large toy while walking. Climbs onto and down from furniture w/o help. -fine-> builds tower of 6 or 7 cubes. Right or Left handed. Imitates circular ad vertical strokes. Scribbles and paints. Starts to turn knobs. Puts round pegs into holes.36 months: -gross-> climbs well, pedals tricycle, runs easily. Walks up and down stairs with alternating feet. Bends over easily w/o falling.-fine-> undresses self, copies circle. Builds tower of 9 or 10 cubes. Holds a pencil in writing position. Screws/unscrews lids, nuts, bolts. Turns book pages one at a time.WARNING (KNOW FOR TEST) -does not point finger at 12 months -after independent walking for several months, it is a concern if: -persistent tip toe walking -failure to develop a mature walking pattern-By 18 months it is a concern if: -not walking -not speaking 15 words -doesn't understand function of common objects-By 2 years it is a concern if: -doesn't use 2 word sentences -doesn't imitate actions -doesn't follow basic instructions -can't push a toy with wheels -By 3 years it is a concern if: -difficulty with stairs -frequent falling -can't build tower of more than 4 blocks -difficulty manipulating small objects -really bad separation anxiety -cannot copy a circle -doesn't engage in imaginary play -can't communicate in short phrases -doesn't understand simple instructions -little interest in other children -unclear speech, persistent drooling

Communication and Language for Toddlers

By 2 should be half understandable. By 3 should be 75% understandable.(Receptive Language): ability to understand what is being said or asked. More advanced than expressive. (Expressive Language): ability to communicate one's desires and feelings. (Echolalia): repetition of words and phrases without understanding. Ex: bananananana(Telegraphic speech): speech with only essential words to get the point across. Ex: Me want cup.12 months: (Receptive)->understands common words independent of context. Follows 1 step command accompanied w/ a gesture.(Expressive)-> Uses a finger to point to things. Imitates or uses gestures like waving goodbye. Communicates desires w/ word and gesture combinations. Vocal imitation. First word. 15 months: (Receptive)-> Looks at adult when communicating. Follows 1 step demand w/o gesture. Understands 100-150 words. (Expressive)-> Repeats words they hear. Babbles in what sounds like sentences.18 months: (Receptive)-> understands the word "no"; comprehends 200 words. Sometimes answers the question, "what's this?" (Expressive)-> uses at least 5-20 words. Uses names of familiar objects. 24 months: (Receptive)-> points no named body parts, points to pictures in books, enjoys listening to simple stories. Names a variety of objects in the environment. Begins to use "my" or "mine". (Expressive)-> Vocab of 40-50 words, sentences of two or three words "me up", and asks questions "what that?" Uses simple phrases. Descriptive words like hungry, hot. 2/3 of what child says should be understandable. Repeats overheard words.30 months: (Receptive)-> follows a series of 2 independent commands. (Expressive)-> vocal of 150-300 words. 36 months: (Receptive)-> understands most sentences. Understands physical relationships (on, in, under). Participates in short conversations. May follow a 3 part command. (Expressive)-> Speech usually understood by family, about half is understood by those outside the family. Asks "why?". 3-4 word sentences. Talks about the past. Vocab of 1,000 words. Can say name, age, and gender. Uses pronouns and plurals. WARNING (KNOW FOR TEST) -doesn't look at person when communicating by 15 mo. -doesn't have 40-50 words by 24 months-less than 2/3 of words understandable-loss of previously obtained speechRemember that children exposed to more than one language may have slightly delayed first word.

Echolalia

Repetition of words and phrases w/o understanding, common in toddlers.

Telegraphic speech

Speech that contains only the necessary words to get the point across, common in toddlers Ex: "Go car

Toddlers need ____ minutes of structured activity and ____ hours of unstructured activity per day

30 minutes of structured, 1-3 hours of unstructured

Toddlers engage in what type of play?

Parallel play

True/False: The best toys for the toddlers are household items, such as stuffed animals or chalk, instead of interactive electronic toys.

True!

An 18 month old needs ____ hours of sleep per day; A 24 moth old needs ____ hours of sleep per day; a 3 year old needs ____ hours of sleep per day

13.5 13 12*most children discontinue daytime napping by 3 years of age

Psychosocial (Erikson) and Cognitive Development (Piaget) for toddlers

Erikson: autonomy vs. shame and doubt -achieves autonomy and self-control -separates from caregiver -negativism, "No!" -labile emotions -cannot take turns in games until 3 years old.Piaget: Sensorimotor and Pre-operational -parallel play -able to use symbols to allow for imitation. -objects begin to have unique characteristics. -dramatic playObjects are "small" or "big", have specific colors, etc. Childproofing is so important especially with toxins because they have no concept that they could hurt themselves.

Social and Emotional Development in toddlers

General Development: -egocentrism -emotionally labile -learns difference between genders. -aggressive behaviors -unclear body boundaries -toilet training Separation Anxiety: -Reemerges at 18-24 months. -24 to 36 months it lessens again. Temperament: -expresses frustration w/ temper tantrums -comfort from routines -ranges from low to moderately active, regular, and predictable to highly active, more intense, and less adaptable. -some cultures feel boys should not cry. Cultural Influences: -homelessness or poverty can affect ability to grow physically and developmentally. -may affect emotional development.

Promoting Healthy Growth and Development in Toddlers

Play:-major socialization medium -parallel play -cannot share -short attention span -adequate physical activity is essential -limit TV and electronic devicesEarly Learning: -talking and singing -repetitive naming of objects -running narrative of activities -reading to toddler crucial Safety: -car safety -toxins -water safety -falls Nutrition: -cultural dietary practices -self-feeding -picky eaters -variety of foods -no need for juice; encourage water (or milk) -oral hygiene -toddlers who consume a strictly vegan diet are at risk for deficiencies in Vitamin D, B12, and iron. Supplementation is needed to promote adequate nutrition and growth. Sleep and Rest: -sleeps through the night -one daytime nap

Signs a toddler is ready for potty training

-regular bowel movements -express need to use the bathroom -diaper is not always wet -toddler can walk well and pull down pants -toddler follows parent to the bathroom -toddler climbs on toilet

Growth and Developmental Changes in the Preschooler

(Weight)-> gains 4 to 5 lbs per year. (Height)-> grows 2.5 to 3 inches per year.Head circumference is no longer measured.

Physiologic Changes as Preschooler Grows

Neurologic System: -continued myelination of spinal cord; bowel and bladder control completed. Cardiovascular System: -heart rate decreases -blood pressure increases. Other organ systems: -continue to grow and matureCommon concerns: -constipation -bedwetting (tends to run in families; usually resolves by 7 years old).

Psychosocial (Erickson) and Cognitive Development (Piaget) for Preschoolers

Erickson: initiative vs. guilt -begin to assert own power and control over the world through directing interactions. -Superego (questions "am I good or bad?")Piaget: Pre-operational Thought -symbolic play -magical thinking -animism -talk about things that happened in the past

Motor Skills for Preschoolers

3 years: (Gross)-> climbs well, pedals tricycle, runs easily, walks up and down stairs w/ alternating feet. Bends over easily w/o falling. (Fine)-> undresses self, copies circle, builds tower of 9 or 10 cubes, holds pencil in writing position. Screws/unscrews lids, nuts, bolts. Turns book pages one at a time. 4 years: (Gross)-> throws ball overhand, kicks ball forward, catches bounced ball, hops on one foot, stands on 1 foot up to 5 seconds. Alternates feet going up and down stairs. Moves backward and forward with agility. (Fine)-> uses scissors successfully, copies capital letters, draws circles and squares, traces a cross or diamond. Draws a person w/ 2 to 4 body parts. Laces shoes. 5 years: (Gross)-> stands on 1 foot 10 seconds or longer, swings and climbs well, may skip. Somersaults, may learn to skate and swim. (Fine)-> prints some letters. Draws person w/ body and at least 6 parts. Dresses/undresses w/o assistance, can learn to tie laces. Uses fork, spoon, and knife (unsupervised) well. Copies triangle and other geometric patterns. Mostly cares for own toileting needs.

What are some gross motor skills expected in a 4 year old?

-hopping on 1 foot -standing on 1 foot for 5 seconds -throwing a ball overhand -kicking a ball forward -alternating feet going up stairs

What are some gross motor skills expected in a 5 year old?

Standing on 1 foot for 10 seconds -skipping -somersalts -swimming -skating

Social and Emotional Development in Preschoolers

General Development: -learn that they are independent beings, and their actions matter. -more aware of peer relationships -proud of accomplishmentsCultural Influences: -affects expected roles and behaviors Fears: -monsters-strangers -loud noises Temperament: -social flexibility -active participant -determines ability to adapt to changeRegression is common if there's stressful stimuli (grandma dies, they don't feel good, etc.)

Promoting Healthy Growth and Development in Preschooler

General: -consistency is key -thrive on rituals and routines -discipline -praise -allow child adequate time to complete tasks Safety: -keep safe from themselves; imagination. -car safety -toxins -water safety -falls Sleep ad Rest: -12 hours of sleep per dayWARNING (KNOW FOR TEST) At 4 years old you should be worried if: -cannot jump (neuropathy? physical?) -cannot stack 4 blocks-cannot copy circle -ignores other children (how's their hearing?)-cannot use "me" and "you" properly -no fantasy playAt 5 years old you should be worried if: -often unhappy or sad -cannot stack 6-8 blocks -cannot brush teeth or wash hands -little interest in playing with other children -cannot use plurals or past tense -aggressive

Communication and Language in Preschoolers

Speaks in complete sentences. Knows names of familiar animals. Knows colors. Can follow a 3 part command. Understands "same" and "different" Talks about past, future, and imaginary events. Can count to 10. By 5 speech is 100% understandable.

Growth and Developmental Changes in the School-aged Child

(Weight)-> 7 pounds a year (Length)-> Grows around 2.5 inches per year. Have increased height by at least 12 inches. (Other growth)-> secondary sexual char. begin to appear in preadolescence. No longer measure head circumference.

Physiologic Changes as School-age Child Grows

Neurologic System: -growth of facial bones changes facial proportions -loose "baby face" appearance Respiratory System: -respirations become diaphragmatic in nature Immune System: -fewer infections -by 10 years at adult level Genitourinary System: -puberty begins in later stageGastrointestinal System: -less caloric need -permanent teeth begin to replace deciduous teeth Musculature System: -improved strength and coordination -this is when you start worrying about obesity bc they have less opportunity to move and run aroundVision: -20/20 visual acuity -peripheral vision, muscular control and color discrimination by 7 years

The average school-aged child needs ____ calories per kilogram daily and _____ grams of protein

70 calories per kg 28 grams of protein

Psychosocial (Erickson) and Cognitive Development (Piaget) for School-Aged Child

Erickson: Industry vs. Inferiority -develop sense of pride in accomplishments and abilities -gain confidence -can develop poor self-confidence -"How can I be good?" Piaget: Concrete operational -lacks abstract thought -learn reversibility -aware that others have own thoughts and desires

Concrete Operational Stage

7-11 years old understands conservation, sees things from other person's point of view, sorts things, thinks through actions

Communication and Language in School-Aged Children

Vocab expands to 14,000 words. More grammatical forms are used. Begin to understand metaphors. Enjoy jokes. Culturally specific words are used.

Social and Emotional Development in School-Aged Children

General Development: -more independent -school is a large factor -organized sports and activities -friends Cultural Influences: -child becomes more aware of differences Fears: -want to do well -want to fit in Temperament: -can affect self-image

Promoting Healthy Growth and Development in School-Age Child

General: -keep child active -obesity 20.7% among 6 to 11 year oldsSleep and Rest:-9 to 12 hours sleep per day -consistent sleep schedule -rest injuriesSafety: -gun, bike, water, and fire safety -abuse-car safety; no front seat until 12 years old or 57 inches -emotional safety (are you safe? depressed?)1. accidents are leading cause of death 5-14 year olds. 2. then cancer in 5-9 year olds and suicide 10-14 yr olds.3. congenital malformations, deformations, or chromosomal abnormalities 5-9 and cancer 10-14.

Growth and Developmental Changes in the adolescent

Hormones stimulate development of secondary sex characteristics. Sexual development categorized in Tanner stages 1 to 5 (1 = baby; 5 = fully grown adult)Period of rapid growth for girls: -rapid height increase prior to menarche-height increase stops for girls 2 yrs. after menarche-2 to 8 inch increase in height -weight increases by 15-55 lbs. Period of rapid growth for boys: -rapid height increase usually starts after girls.-height increases 4 to 12 inches -weight increases 15 to 65 lbs.

Girls enter puberty around ___ years old, and boys enter puberty around ____ years old.

Girls 9-10; Boys 10-11

Physiologic Changes as Adolescent Grows

Neurologic System: -faster neural processing Respiratory System: -respiratory rate reaches adult rate. -deepening of voice.Integumentary System: -skin becomes thick and rough -increased sebum leads to acne -increased sweat production Gastrointestinal System: -full set of permanent teeth -wisdom teeth Musculature System: -muscle mass and strength increase -growth plates close as puberty ends

Psychosocial (Erickson) and Cognitive Development (Piaget) for adolescents

The adolescent is very idealistic, constantly challenging the way things are and wondering why things cannot change. This leads to the adolescent's feeling of being omnipotent.Erickson: identity vs. role confusion -develop own sense of self -develops role confusion if not able to develop own sense of self -revisits previous stages of development -need to be independent (w/ hands off guidance)Piaget: Formal Operational Thought -progresses from concrete to abstract thought -develops ability to think outside present into various concepts -early adolescence-> egocentric (center of everyone's attention)

Motor Skills in Adolescents

Gross Motor Skills: -develop endurance -can follow complicated instructions -speed and accuracy improves Fine Motor Skills: -computers increase fine motor skills-precise hand eye coordination and dexterity

Language and Development in Adolescents

Language: -use correct grammar and parts of speech -use of colloquial speech (slang) increases -comparable to adults by late adolescenceEmotional and Social: -parent and peer relations -body image -sexuality -dating

Promoting healthy growth and development in adolescent

Physical activity: -team sports -injuries Learning:-large part of life -cultural and socioeconomic considerations Sleep and Rest: -9 to 10 hours of sleep per day -more awake at night, sleep later Safety: -motor vehicles -poisoning (drugs) -accidents -gun safety -piercings and tattoos Nutrition: -increase calories during growth -need calcium, zinc, and iron-risk of obesity

Health Assessment of Children

1. Preparing for health history 2. Health history 3. Physical exam -general appearance -vital signs (temp, BP, pulse, RR, O2 sat) 4. Pain Assessment 5. Body Measurements 6. Physical Assessment 7. Developmental Screening

Preparing for Health History

1. gather supplies -consider pt. age w/ approach 2. approaching caregiver -det. who adult is; avoid judgement. 3. approaching child -age appropriate 4. observe interactions -2 way communication? -parents comments 5. assess type of history needed -comprehensive vs. problem focused*Involve child when appropriate *Observe parent-child interactions *Ask open-ended questions *Help caregiver to stay on track

Health History

1. demographics 2. chief complaint and HPI 3. past health history and family health history4. review of systems -growth and development -also more traditional questions5. developmental history -milestones ass.w/ age6. functional assessment -should contain info. about child's daily routine -using comfort item like blanket or Paci? 7. family composition, resources, and home environment -who is helping raise the child; foster care? -access to meds? doctor? to variety of foods? -where do they live? apartment, homeless, etc. -smokers in house?

Newborn Health Assessment

-vital signs-check fontanelle; should be flat. -sunken-> dehydrated; bulging-> meningitis-neck-> check lymph nodes-while quiet listen to heart and lungs -bob head for HR; 1 minute -feel abdomen for respirations (diaphragmatic breathers)-cap refill <3 seconds-feel pedal and popliteal pulses -inguinal area (nodes may be palpable)-take off diaper and check genitalia -circumcised? -labial adhesions?-check the rectum is open -check for hip dysplasia (feel hip click) -check ears then mouth-check gag reflex and while looking in mouth look for: -thrush -lesions in the mouth -look at spine for sacral dimpleDo a cuddle test! Pick up the baby and if they are floppy/rigid that's worrisome. If they cuddle in to you that's good.

Place to Perform Exam and Exam Direction

Newborn: (Place to perform)-> lie on exam table or in caregiver's lap. (Exam Direction)-> running dialog w/ caregiver, explain steps along the way. Infant: (Place)-> caregiver's lap or exam table with caregiver right beside. (Direction)-> explain each step to caregiver, address child by name, invasive parts last. Toddler: (Place)-> freedom of movement if possible. May stand between caregiver's legs or sit on lap. (Direction)-> introduce self to caregiver and child. Explain most steps to the child. Allow child to handle instruments. Invasive last. (start w/ heart and lung sounds while they're being quiet). Preschool: (Place)-> may be willing to sit on exam table with caregiver close by. (Direction)-> allow child to decide order of exam; explain what instruments do and let child try them. Speak to caregiver before and after exam.School-Age: (Place)-> sitting on exam table where they have eye contact w/ caregiver (Direction)-> Include child in all parts of the exam. Use head to toe approach w/ genital exam last. Speak to caregiver before and after. Early Teen: (Place)-> some may be willing to have caregiver wait outside exam room. (Direction)-> speak to child using mature language; appeal to their desire of self care. Head to toe. Genital exam last. Late Teen: (Place)-> explain to caregiver that teen needs privacy and ask them to wait outside room. (Direction)-> explain confidentiality to caregiver and teen; allow time talking w/ them together and separately. Head to toe. Genital exam last.

General Appearance

1. First impressions count! -assess skin color, respiratory effort, speech, attitude2. Is child ill or well? 3. Child's expression and energy level -toddlers usually won't let you touch them4. Posture and development -be aware what's normal for age group and what isn't

Vital Signs

Temperature (axillary) Pulse Respiratory rate Oxygen saturation Blood pressure Pain assessment Body measurements -height, weight, head circumferenceMonitoring equipment

Taking Temperature in pediatric patients

Use least invasive method. Most often axillary or oral. Rectal may require an order. Have you had anything to eat or drink recently?Any popsicles? Write measurement and route taken.

Taking BP in pediatric patients

Always use appropriate cuff size!All infants and children have their BP checked w/ vitals. On outpatient basis, kids 3 yrs + should have at least one measurement yearly. -can be done on arm or leg-often on legs, especially w/ infants and young kids -leg controlled better ALWAYS NOTE MAP - MEAN ARTERIAL PRESSURE

Pulse and Respirations in pediatric patients

Check apical pulse under 2 years old for full minute. Count respirations for full minute, remember the infant's breathe diaphragmatically so count abdominal movements. Infants: -HR: 80-150 -RR: 25-55 Toddler: -HR: 70-120 -RR: 20-30 Preschooler: -HR: 65-110 -RR: 20-25 School-age: -HR: 60-100 -RR: 14-26Adolescent: -HR: 55-95-RR: 12-20Heart rate goes up ad down with their breathing

Oxygen Saturation in pediatric patients

Placement! -can be placed on finger, toe, ear, foot, or forehead. -site needs to be changed every 4-8 hours -depends on facility

Pediatric Pain Assessment

5th vital sign use age appropriate pain scale

Pediatric Body Measurements

Length or Height -lying position until 24 months -standing when they can stand independently and cooperate Weight -lying or sitting for infant and toddler -standing as age -infants weighed w/o clothing or diaperHead Circumference -measure at well check and hospital admission -done until 3rd birthday

Physical Assessment for pediatric patients

Skin -color (mottled, jaundice, pale, etc.)Hair and nails -gives idea of general healthHead -note if fontanel open or closed -anterior fontanel should close at 18 mo. -head control -look at shape: flat on back-> suggest tummy timeNeck -neck control -supple? -move easily -palpable lymph nodes?Eyes -following objects -pupil responses -strabismus -equal light reflex -look for ocular tumors (red eye reflex) -crossed eyes normal until 4-6 months -cataracts? -pupils should react like adults and follow a fingerEars -note low-set ears, skin-tags on ears, pits or dimples. -low set could mean genetic condition -ear tags/pits -> look at kidneys! -pull down on earlobe under 3 years -pull up and out on older childrenNose and sinuses -persistent foul drainage could mean foreign body -infants under 3 mo. obligate nose-breathers -want to keep as clear as possible -cartilage is there -sinuses not well developedMouth and throat -tight lingual frenulum (tongue-tie) -note teeth and dental caries. -tongue angle can affect breast feeding -frenulum may be snipped to allow for better feedsThorax and Lungs -Pectus Excavatum -Pectus Carinatum Breasts -Tanner staging -Development begins 2 years prior to period -check for lumps and bumpsHeart and Peripheral -murmurs not unusual -variation of heart rate w/ inspiration -pulses measured in brachial area under 2 yr. -cap refill time should be <3 seconds -check for perfusionAbdomen -umbilical stump (dry, no drainage or redness) -umbilical hernia -can palpate w/ childs' hand over yours to reduce tickleMusculoskeletal -observe spine for scoliosis, ROM, strength -hip dysplasia, normal for legs to be bowed in infancy -observe walking gait -count and examine fingers and toesNeurologic -LOC -balance and coordination -sensory testing -reflexes -jump; stand on 1 footGenitalia and Anus -Penis: -circumcised or not? -foreskin retracts (DON'T FORCE) -scrotum: 2 testes? -any signs of hernia? -Tanner staging -Female External Genitalia: -Tanner staging -pubic hair at inappropriate age? -labial adhesions? -vaginal discharge- pinkish d/c after birth normal

When is the best time to do a physical assessment on an infant?

1-2 hours before feeding

Physical Assessment of Infant Considerations

-auscultate heart, lungs, and abdomen first -undress down to their diaper -warm the stethoscope -leave traumatic procedures to last -allow parent to be nurturer

Physical Assessment of Toddler Considerations

-assume eye level and talk quietly -allow child to sit in parent's lap -use little touch in beginning -incorporate play -introduce equipment slowly -let toddler touch and hold equipment

Physical Assessment of Preschooler Considerations

-allow child to "help" -offer options -explain procedure using dolls -compliment child on their cooperation

Physical Assessment of School-Aged Child Considerations

-provide honest information using concrete words -allow child to help -allow child to wear underpants under gown -respect child's desire to avoid pain or insult -privacy and respect important

Developmental Screening

Brief assessment that identify children who warrant more intensive assessment and testing. KNOW FOR TEST These are screening tools, NOT diagnostic.

Pediatric Pain

Highly individualized. Children may lack verbal capacity to describe their pain. Unmanaged pain can lead to physical and emotional consequences.

Factors Influencing Pain

Age and Gender: -pain transmission and perception is present before birth. -preterm to children of any age can experience pain; may feel MORE pain r/t inability to buffer. -may be difference in how boys and girls experience pain, more research is warranted. May be r/t genetics, hormones, family and culture. Cognitive Level: -as child's understanding of pain increases this impacts their choices for coping strategies. -as cognition increases, better able to express information about pain. -developmental delays affect communication. Previous Pain Experience: -number of episodes, type of pain, severity, effectiveness of pain treatment, and previous response of child affects how current experience is perceived. -ex: kids with sickle cell are used to a lot of pain so they don't show how much they're in-severe pain in neonates or young infant can lead to sensory disturbances and altered pain response into adulthoodFamily and Culture Temperament Situational Factors

True/False: Preschoolers may not verbally report their pain because they think adults are aware of it.

True

True/False: Adolescents can deny or refuse pain medication.

True

Developmental Considerations r/t pain in pediatric patients

Infants: -preterm have greater intensity of pain -facial grimacing -brow contracting, chin quivering, body movements-crying, eyes tightly closed, stiff bodyToddlers: -intense emotions and upset-aggression and physical resistance-may try to hide or leave room-age appropriate word like "boo-boos"-act out to show pain bc they don't know how to express themselvesPreschoolers: -quiet and withdrawn -can better describe pain-magical thinking -relate things that don't make sense -(ex: my ear hurts bc I didn't clean my room) School-Age Children: -may deny pain -fear and embarrassment r/t pain - acknowledge and validate their pain-may fear being embarrassed by acting out behaviors in response to pain, like screaming or thrashing-typical response might be to withdraw by staring at the television-may be more worried about illness and what it means than actual pain.Adolescents: -concern over body image and losing control-worry about their own reaction to the pain-let them know everything is confidential-denial or refusal of medications-their mood and what they think is expected of them will affect their response to pain

Pain Assessment QUESTT

Question the child.-what does pain mean to the child?-use words child can comprehend more easily-inquire about similar experiences in the past and how they dealt w/ it.-Determine if they told others they were in pain and how they conveyed that message (crying, acting out, or pointing to hurting area)-review history of pain-circumstances in which the child feels the pain-cultural aspects, caregiver's attitude and expectations-previous experiences -any education or teaching r/t pain management -continue to form questions to ascertain the following: 1. location, quality, severity, and onset of pain. -also circumstances kid experiences the pain -have child point to area on themselves or a doll2. conditions, if any, that precede onset of pain -and conditions that followed onset of pain3. any associated sx like weight loss, fever, vomiting, or diarrhea which may indicate current illness4. Any recent trauma including any interventions that were used in an attempt to relieve pain.Use reliable and valid pain rating scale.- establish a baseline to better determine effectiveness of interventionEvaluate behavior and physiologic changes. Secure caregiver's involvement.Take cause of pain into account when interviewing.Take action and evaluate resultsDOCUMENT

Pain Rating Scales

Standardized rating scales means they are tested and proven accurate. Self-report should be used in conjunction w/ observation and discussion when age-appropriate. -be situationally aware -preschoolers/others may want to please you by saying pain is less than it is

CRIES Scale

Pain scale used for neonates, measuring crying, oxygen requirements, vital signs, expression, and sleepiness.-for neonatal postoperative pain assessmentBehavioral assessment tool that includes behavioral and physiologic parameters. Level 0: -no crying -doesn't require O2 -normal HR & BP -normal expression -no sleeplessness Level 1: -high pitched crying - <30% FiO2 needed -increased HR and BP <20% -grimacing -wakes frequently Level 2: -inconsolable crying - >30% FiO2 needed -increased HR and BP >20% -grimacing and grunting -awake constantly

FLACC Scale

Pain scale used for ages 2 months- 7 years measuring face, legs, activity, cry, and consolability.-Behavioral scale for painNonverbal young children and children with cognitive impairment. It has been demonstrated to be reliable tool for children from age 6 mo. to 7 years of age. Includes additional descriptors of behaviors most commonly ass.w/ pain that have been validated in children w/ cognitive impairment.1. Face 2. Legs 3. Activity 4. Cry 5. Consolability

FACES Pain Rating Scale

Self report tool used for kids 3 to 8 years oldranging from 0 - 10

Numeric Pain Rating Scale

Self-report pain measure where a client verbalizes a number to correspond with a perceived level of pain from no pain to most severe pain-8 years and older Pain is what the patient says it is, not what you think it is! Ask qualifying questions

Nonpharmacologic Pain Management

Require child to focus on specific area rather than the pain. Strategies help to change the interpretation of the painful stimuli, reducing pain perception or making pain more tolerable.Also help to decrease negative attitudes, thoughts, and anxieties which improves the child's coping mechanisms. Sucking and sucrose. Behavioral-Cognitive Strategies: -relaxation -distraction (bubbles) -imagery -biofeedback -thought stopping -positive self talkBiophysical Intervention: -sucking and sucrose (calms and soothes); opioid effect-heat and cold applications; they distract, cold also numbs-massage and pressureNurse's Role: -help child and family choose best method for them-chronically ill pt. will have a pain action plan-assist parents and child to use technique

Signs for Chronic Pain in Children

-sleep disturbances -irritability -exhaustion -depression -changes in eating patterns -mood disturbances

Pain management techniques for children

-Use non-pharmacologic AND pharmacologic measures -individualize interventions based on amount of pain and characteristics of child -educate child and family about pain relief interventions

Pharmacologic Pain Management

-nonopioid analgesics -opioid analgesics -adjuvant drugs -local anesthetics

Drug Administration Methods

-Oral -Rectal (when meds can't be taken PO) -IV -Patient-controlled analgesia -local anesthetic application -epidural analgesia -moderate sedationEMLA is common choice for effective, painless local anesthesia. Achieves anesthesia to depth of 2 to 4 mm. Requires 60 to 90 minutes application time. Reduces pain at site for up to 24hrs after injection. Approved for infants 37 wks gestation or older. LET or TAC used for lacerations.

Nurse's role in managing pain

- Use topical anesthetic (EMLA) at site of skin or vessel puncture- Use Nonpharm and Pharm strategies for pain relief- prepare child/family ahead of time about procedure- use therapeutic hugging to secure child- use smallest gauge needle possible- coordinate care so several tests can be performed from one sample if possible- use kangaroo care (skin to skin) for newborns before and after heel stick. -provide NNS, w/ sucrose solution, if appropriate; pacifier; or breastfeeding for newborns several minutes before the procedure.- opt for venipuncture in newborns instead of heelsticks

Nurse's role in managing chronic pain

Patients w/ chronic pain experience significant: -physical -emotional -social consequences Children w/ chronic pain have increased risk of: -depression -suicidal ideation -problems w/ school functioning -predisposition of chronic pain in adulthoodNurses need to: -assess level of stress on pt. and family -get support departments involved

Nutritional Requirements

Normal nutritional requirements differ based on age, developmental stage, weight, and level of activity. -infants -toddlers -preschoolers -school-age -adolescents

Infant Nutrition

-encourage breastfeeding until 6 mo. minimum-formula feeding is individual decision -at age 6 mo. assess readiness for solid foods -coordinated chewing and swallowing -new foods introduced every 4-7 days: allergy recondition-Foods should be introduced in this order-> fruits, veggies, meat, eggs, and citrus-At this time rice cereal, baby foods, and finger foods can be introduced -ask what else they add to bottle other than milk-at age 1 whole milk can be added (too much fat before that age)

When and how should a parent assess a baby's readiness for solid foods?

6 months- introduce a new food every 4-7 days, leaving eggs and citrus to last.

Breastfeeding

Recommended in all infants including sick or premature. Breast milk includes lactose, lipids, polyunsaturated fatty acids and amino acids. The concentration of iron in breast milk is less than that of formula, but has increased bioavailability and usually meets infants requirements for the first 4 to 6 months. Mom's diet must be balanced for the milk to provide the nutrients the baby needs. Bottle feeding often promotes over feeding. For every age in month, they should be eating that in ounces.

Breastfeeding should be encouraged until at least ____ months of age.

6 months

Toddler Nutrition

Establish healthy eating habits early. Weaning breastfeeding is an individual decision but the usual age to wean is around 1 year. Weaning from the bottle around 12-15 mo. and make sure they have adequate calcium.-also due to teeth -DO NOT put to bed w/ a bottle Toddlers are picky eaters- DO NOT substitute unhealthy foods "just so the child will eat"-room temperature bite-sized pieces -encourage self feeding and encourage water drinking

Preschooler Nutrition

•Quality of food consumed is more important than quantity. Primary teeth present at this time.•Continue to learn & build on healthy eating habits•Erratic eating•Parents - maintain positivity & patience•Offer choices•Do not substitute for unhealthy foods if not eating as muchObesity is a concern at this age group (13.9%)-> can develop fatty liver, diabetes, etc.

School-age Nutrition

-children should be consuming decreased calories -common influences: family, friends, and media -obesity is more prevalent during this age group -limit fat and processed sugar -not recommended to use food as a reward -involve them in healthy choices*Calorie needs decrease, obesity increases -> limit fats and processed sugars, do not use food as a reward

Adolescent Nutrition

-desire for independence-lots of educational support needed at this time -there is a need for zinc, calcium, and iron-increased calories needed -(growth and sexual maturation)-treating disorders support needed at this age group*Calorie needs increase; need for zinc, calcium, and iron

Eating Disorders

Anorexia Nervosa-> body image disturbance, extreme weight loss.Bulimia Nervosa-> cycle: normal food intake, purge. Complications: F&E imbalance, decreased blood volume, cardiac arrhythmias, esophagitis, rupture of the esophagus or stomach, tooth loss, and menstrual problems. Nursing Considerations: -restoration of F&E balance (slow process) -encourage family involvement -prevent complications -referrals to resources and support

Failure to Thrive (FTT)

Inadequate growth resulting from inability to obtain or use calories required for growth. -can be organic or inorganic -little to no weight gain **less than the 5th percentile for weight/age Nursing Management: -education -thorough assessment -role model -increased calories for catch up -multidisciplinary care -documentation of progressionKeep in mind that the pt. may not look malnourished.Growth chart isn't progressing well.

Common Food Allergies

-cow's milk-eggs-wheat-peanuts-soy-fish and shellfishLook for hives-> may be larger response next time.

Optimizing nutrition

Overweight = BMI at or above 85th percentileObese = 95th percentileTwo part issue-> diet and exercise

Nutritional Assessments

Growth measurements Plot growth rate Nutritional history

Providing Nutritional Support

Enteral nutrition: -routes: NG, NJ, OG, G-tube-functioning GI tract but cannot ingest nutrients orally-cost effective, safer than parenteral nutrition -bolus or continuous feeds done to train stomach for normal feeds -special consideration -correct measurement -placement verificationParenteral Nutrition: -TPN (Total Parenteral Nutrition) -highly concentrated solution of carbohydrates, electrolytes, vitamins and minerals -TPN is given centrally, not peripherally -long term use not typical -complications include air embolism, electrolyte imbalance, infection

TPN is given ____ (centrally or peripherally).

Centrally

Communicable Diseases

Illnesses passed by indirect and direct contact. Risk factors: -immature immune system -limited prior exposure to communicable diseases -poor health/immunodeficiency Routes of Transmission: -air borne -droplet-contact (esp. in kids bc they put everything in their mouth)How to Prevent Spread of Infection: -handwashing (yours and the Childs)-standard precautions -avoid exposure to infected people -promote immunizations -decrease/eliminate pathogensClinical Manifestations: -non specific symptoms: fever, fatigue, weight loss, myalgia, decreased appetite, nausea, vomiting, diarrheaDiagnostics: -cultures: bacterial, fungal, or viral Clinical Therapy: -antipyretics (Tylenol-use appropriate dose!)-antibiotics (beware of resistance!)-antiviral medications (not often used; ex: Tamiflu- causes GI upset)

Mechanism of Immunizations

Active Immunity: Antibody production stimulated by vaccine antigens w/o causing clinical disease. Passive Immunity: Antibodies produced in another host (human or animal) given when child needs antibodies faster than the body can make them -includes immune globulins -does not confer lasting immunity; child will need vaccine in the futureContraindications to Vaccine Administration: -history of anaphylactic reaction to the vaccine or one of it's components -hx of encephalitis within 7 days of DTaP-delay if moderate to severe acute illness (don't give if they already have a fever) -live virus vaccines aren't given to children who are immunocompromised or receiving chemotherapy.How can pain be minimized?-provide local anesthetic -give child as much control as possible -be honest with them -provide sucrose drink and pacifier Where should it be given? -IM is the most common route in kids -some vaccines are administered subQBarriers to Immunization Compliance? -economic factors -limited access to health care -lack of convenient primary care -parental knowledge deficit -religious/cultural prohibitions -current trends

Active vs. Passive Immunity

Active = body makes it's own antibodies Passive = antibodies produced by another host and given to someone

Contraindications to vaccines

-prior anaphylaxis reaction -history of encephalitis w/in 7 days of dTap vaccine -moderate to severe acute illness -immunocompromised or receiving chemo means child cannot get live vaccines

Strategies to minimize pain during vaccine admin in child

-provide local anesthetic -give child as much control as possible -be honest -provide sucrose drink and pacifier

DDST (Denver Developmental Screening Test)

Recognizes possible developmental delays in children which require referral.-normal DDST involves no delays, and 1 or less cautions -suspect DDST involves 2+ cautions or 1+ delays

Pediatric Hematologic Considerations (Liver)

Production of blood cells begins around 8 weeks in-utero.Fetus receives iron (Fe) through the placenta from Mom. Erythropoietin is derived primarily from the liver in the fetus, kidneys take over at birth. 3 types of normal hemoglobin present at birth: -adult (Hgb A and A2) present in utero and predominate until around 2 months after birth-Hgb F (fetal) predominate at 8 weeks GA, decreases to around 70% by birth and disappears by 6-12 months

Production of blood cells begins at about _____ weeks in utero

8

Hgb F (fetal) predominates at ___ weeks gestational age, decreases by ____ percent by birth, and disappears by ___ months

8 70% 6-12

Iron-deficiency Anemia

Etiology:-Low hemoglobin production due to low iron-At risk infants are those who are born premature, exclusively breast-fed past 4 months of age w/o Fe supplementation, low socioeconomic status, and exposure to lead.-Infants and children who drink cow milk w/o adequate Fe intake are at increased risk.Diagnostic Criteria: -For children 6 mo.-5yrs: -Ferritin <15 mcg/L AND Hgb <11 g/dL-For children 5-12 years: -Ferritin <15 mcg/L AND Hgb <11.5 g/dLClinical Manifestations: -growth retardation in chronic cases may occur -irritability, HA, weakness, SOB, dizziness, pallor, and fatigue -assess for spooning of nails, splenomegaly, and flow murmur Nursing Interventions: -determine and treat underlying cause -avoid NSAIDs, only take Tylenol-limit cow's milk intake to under 24 oz./day -Fe supplementation -start at 4 mo. for breast-fed infants -may turn stools black in color -may cause constipation*Iron drops should be placed behind the tongue in infants and given via straw in older children to prevent teeth staining- take 1 hr before meal! -dosing: 3-6 mg/kg/dayNursing assessment: -thorough history -check for spooning of nails -inspect for pallor -auscultate for flow murmur -assess for splenomegaly

Diagnosing Iron-Deficiency Anemia in a 4 year old would include which blood test findings?

-Ferritin < 15 mcg/L -Hgb < 11 mcg/L

Sickle Cell Disease

Inherited hemoglobinopathy that causes hemolytic anemia and vaso-occlusive phenomena-genetic disorder (autosomal recessive) that results in abnormal crescent shaped, sticky RBC's.Pathophysiology: -char. by hemolytic anemia and vaso-occlusive phenomena -Sickling RBC's --> increased blood viscosity --> RBC's cannot pass through capillaries/venules -->local tissue hypoxia --> ischemia --> infarction-triggering events may be stress, cold, fever, illness, hypoxia -most severe type is HgbSS Diagnostics: -assessed for newborn screening in all 50 states -electrophoresis confirms the diagnosis -infants are asymptomatic until 3-4mo. due to presence of fetal Hgb Nursing Assessment: (skin): lesions, pallor, ulcerations, breakdown, jaundice, pallor and dry/cracked mucous membranes, scleral icterus (palpate joints): warmth, tenderness, alterations in ROM(palpate abdomen): tenderness, organomegaly (auscultate): adventitious heart and lung sounds -always assess and OLDCARTS- vital signs -> fever, tachycardia/pnea, and hypotension Complications: (dactylitis): vaso-occlusive pain in the small bones of the hands and feet; severe edema, ischemia, and erythema(acute chest syndrome): fever, chest pain, cough, hypoxia, and respiratory distress in setting of lung infiltrates (splenic sequestration): acute drop in blood volume caused by pooling of blood in the spleen; may lead to splenomegaly and shock Nursing Care: 1. medications to prevent infection and vaso-occlusive crises (Hydroxyurea- increases concentration of fetal Hgb, leads to decrease in sickled RBCs; RBC hydration and flexibility is increased).2. management of vaso-occlusive crises which depends on what body areas are affected but common interventions are:-pain management -heat therapy; do NOT use ice packs or cold therapy -promote cellular viscosity by admin IVFs at 1.5-2x maintenance rate -supplemental O2 to prevent hypoxia -manage underlying condition-NO ICE PACKS-educate on signs of vaso-occlusive events

What causes vaso-occlusion in Sickle Cell Disease?

RBCs are sticky, crescent shaped, and have an average lifespan of only 20 days -> stress/trauma leads to sickling RBCs which leads to RBCs getting stuck, causing local tissue hypoxia, ischemia, and potential infarction

What is used to confirm diagnosis of Sickle Cell Disease?

Electrophoresis

Hemophilia

Defect in factor VIII or IX preventing secondary hemostasisHereditary disease where blood does not coagulate to stop bleedingPathophysiology: - X linked recessive disorder -inherited deficiency of factor VIII (Hemophilia A) or factor IX (Hemophilia B) -secondary hemostasis is prevented -> "jelly-like" and unstable clots -> spontaneous bleeding Clinical presentation: -pt. w/ severe form of disease (B) are more likely to have spontaneous bleeding -mild to moderate disease bleeding occurs usually in response to injury/trauma or surgery-bleeding following inciting event may be immediate or delayed-common bleeds in infants: heel sticks, extra cranial sites (cephalohematoma), venipuncture sites, and following circumcision -common bleeds in toddlers, younger children: forehead hematoma, joints -common bleeds in older children: joints, muscles, GI tract, CNS, mouth, and nares -hemarthrosis -> hemorrhage into a joint space Nursing Care: -admin. factor replacement as needed -desmopressin may be admin. to promote release of factor 8 in pt. w/ mild hemophilia A-for bleeding inside a joint: rest, ice, compress, elevate (RICE), immobilization or splinting -avoid NSAIDs, give Tylenol or opiates if indicated -apply pressure to external bleeds -educate and provide support -pt. should avoid activities w/ likelihood of injury -regular exercises and activity to strengthen joints and muscles-soft toothbrush, good oral hygiene -teach caregiver how to give factor replacement

Hemophilia A vs. B

A: deficiency of Factor VIII B: deficiency of Factor IX

Primary Immunodeficiencies

Mostly hereditary or congenitals/sx: failure to thrive (FTT), frequent infections or illnesses, positive family hx.Nursing Care: -infection prevention measures -amin. prophylactic ABX as ordered -admin of IVIG -care before/following hematopoietic transplant

Secondary Immunodeficiencies

Result from chronic illness, malignancy, immunosuppressive medications, malnutrition, prematurity, or protein losing state.

HIV

Secondary immunodeficiency resulting from viral infection, CD4 cells are targeted.Pathophysiology: -exposure occurs 1. most commonly for peds pt. through vertical transmission (mother-to-child transmission may occur at any time during gestation and delivery, and thought breast feeding in the postpartum period) 2. sexual contact 3. IV drug use 4. infected blood products -virus infects CD4 T Helper cells, rapidly replicates -> cell dysfunction occurs-immune suppression increases susceptibility to pathogens both ordinary and opportunisticDiagnostic criteria: -Enzyme-linked immunoassay (detect transplacentally transferred maternal HIV antibodies until 18 months of age)-Western blot (Virologic testing for infants under 18 mo. Utilizes assays [nucleic acid amplification] that detect HIV DNA and RNA. Repeat testing routinely needed as sensitivity increases with time).Clinical Manifestations: (Acute Phase): lymphadenopathy, fever, sore throat, headache, rash, diarrhea, myalgia/arthralgias(Chronic w/o AIDS): recurrent candidiasis [HSV, HPV, Varicella], generalized lymphadenopathy, night sweats, and fatigueNursing Care: 1. Education and prevention -early ID of HIV in pregnant women through universal prenatal screening -provision of adequate prenatal care for HIV + women -maximal reduction of maternal viral load through appropriate use of antiretroviral drugs -ART infants born to HIV + mothers until 6 weeks of age -supplement feeding and maternal support for viral suppression 2. Prophylactic care -infection prevention measures -admin. ABX prophylaxis if indicated -regular PCP visits (keep immunizations UTD)-check in w/ specialists 3. Provide support-counseling, support groups -financial strains- cost of meds and specialty care

How do you test for HIV in children <18 months old?

Virologic testing (assays)

How do you test for HIV in children >18 months old?

Western Blot & Enzyme- linked immunoassays

For infants born to HIV+ mothers, how do we prevent HIV infection?

Antiretroviral therapy (ART) until ~6 weeks of age

Juvenile Idiopathic Arthritis

Etiology: -autoimmune disorder where autoantibodies primarily target joints; three types: 1. Polyarticular- involvement of 5+ joints, often symmetrical involvement 2. Oligoarticular (Pauciarticular)- involvement of 4 or fewer joints3. Systemic- fever, rash, in addition to joint involvement Diagnostics: -diagnosis of exclusion -onset before 16 years, > 6 weeks and unknown etiologyClinical Manifestations: -infant/nonverbal children: irritability, prolonged high-fevers, withdraw from play, difficulty getting up in the morning-joint edema, redness, warmth, tenderness, stiffness -limping, guarding extremity -systemic: red, macular rash Nursing Care: -promote and maintain mobility -heat therapy -splinting as needed -NSAIDs-medication administration: -glucocorticoids (prednisone) -antirheumatic drugs (Etanercept, Methotrexate)

In what order would you do these 4 things: -administer abx -administer Tylenol-access port -take cultures and labs

1. Access the port 2. Draw blood cultures and other labs 3. Administer antibiotic 4. Give Tylenol

Malignancies in children vs. adults

Adults: -arise from epithelial cells (carcinomas) -most commonly involve organs -may be detected earlier r/t screening mechanisms -development r/t lifestyle and environment -longer latent period -metastasis often present at dx Children: -develop from embryonal tissue -cellular growth and development are central mechanisms in pediatric cancer -involves tissues -most often discovered incidentally -develop not significantly influenced by environment and lifestyle factors -shorter latent period -metastasis often present at dx -more responsive to tx than adult cancers

Which of the following is not consistent with malignancy in children? A) the majority of malignancies develop from primitive embryonic tissue B) The development of malignancies is strongly influenced by environmental factors C) Relatively short latency period D) Metastasis is often present at diagnosis

B) The development of malignancies is strongly influenced by environmental factors *children haven't been around long enough to develop any cancers r/t environmental exposure

Detecting Pediatric Cancer

Difficult to detect most pediatric cancers early as s/sx are non-specific w/ insidious onset, and often resemble more common pathologies. Early warning signs for most ped. cancers include: -unexplained pallor and loss of energy -unusual lump, mass, or swelling-sudden, unexplained weight loss -persistent, unexplained fever or illness -easy bruising or bleeding -prolonged/ongoing pain in 1+ body areas -limping, refusal to bear weight -frequent headaches, esp. if in AM or ass.w/ vomiting -eye or vision changes

Cancer in children involves primarily the ________, while cancer in adults involves primarily the ________.

tissues; organs

Common Childhood Blood/Lymphatics Cancers

Leukemias (ALL most common, AML):1. immature lymphoblasts replace normal cells in the bone marrow -> bone marrow cannot maintain RBC's, WBC's, and PLT's -> pancytopenia (anemia, neutropenia, thrombocytopenia)2. Present most often w/ unexplained and persistent fevers, fatigue, weight loss, lymphadenopathy, bone and joint pain, hepatomegaly and splenomegaly Hodgkin Lymphoma: -most common in teens, young adults -malignant cells proliferate in lymph tissues -> lymphadenopathy -> compression of nearby structures, killing healthy cells and invading surrounding lymph tissue before non-adjacent lymph tissues-hallmark is presence of Reed-Sternburg B cells-many cases are ass.w/ Epstein-Barr virus infection -s/sx: weight loss, fever, night sweat, anorexia, fatigue, enlarged and matted lymph nodes (supraclavicular and cervical most common), may be present w/ mediastinal mass Non-Hodgkins Lymphoma:-includes all lymphomas except Hodkin -quick progression so s/sx appear early on in the disease process -tumors can grow anywhere lymph tissue is present -s/sx: pain, lymphadenopathy, abdominal mass which may compress SVC -> superior vena cava (SVC) syndrome

What is leukemia?

Cancer of the WBC's causing uncontrolled proliferation of immature WBCs in the bone marrow, resulting in decreased RBCs, platelets, and normal or increased WBCs.Clinical Manifestations: -bone pain -swollen gums -N/V -recurrent infections -decreased Hgb and platelets -anorexia/weight loss

Continued unexplained weight loss is especially concerning in regards to which type of cancer?

Leukemia

ANC of less than _____ indicates neutropenia.

500

Hodgkin's versus Non-Hodgkin's Lymphoma

Non-Hodgkin's can arise anywhere; Hodgkin's usually begins in the neck, armpits, or chest area.

Autologus vs Allogenic stem cell transplant

Autologous uses child's own stem cells. Allogeneic uses a donor's stem cells/

Hyperleukocytosis

WBC > 100,000, increased viscosity and thrombi, SOB, tachypnea, ataxia, confusion.

Tumor Lysis Syndrome

Decomposing cancer cells release uric acid, potassium, calcium, and phosphate -> call provider if you suspect!!s/sx: -fatigue -muscle cramps -mental status changes -abnormaal EKG / arrhythmia -no urine output

Pediatric Brain Cancers

Medulloblastoma: -cerebellum -s/sx: increased ICP (HA, vomit, dizzy, visual disturbance, ataxia), seizures, weakness, behavioral or developmental issuesHigh-Grade Gliomas: -rapidly progress and become diffuse in brainstem-s/sx: (similar to medullablastoma), HA, seizures, memory loss, language deficits, changes in cognition and behavior

Pediatric Bone Cancers

Osteosarcoma: -often occurs in long bones like the femur, tibia, and humerus -s/sx: intermittent pain localized to tumor sites, changes in gait, limping, soft tissue mass on exam that is tender to palpation-symptoms tend to occur following trauma or injuryEwing sarcoma: -most often occur in the long bones, femur, tibia, fibula -s/sx: intermittent pain that transitions to constant over weeks to month(s), pain worsens w/ activity and at night, edema and erythema over the mass is common-sx often preceded by trauma or injury

Other pediatric cancers

Retinoblastoma: -s/sx: leukocoria (red reflection in eye), strabismus, nystagmus Neuroblastoma:-originates from neural crest cells during fetal development -may arise anywhere throughout the sympathetic nervous system, adrenal most common followed by abdomen-s/sx: contender abdominal mass &/or symmetry, abdominal pain, constipation, back pain/weakness from spinal cord compression, fever, weight loss, anemia, bone pain, bowel/bladder dysfunction, if metastasis to skull will have facial edema and ecchymoses above the eyesWilms tumor: -most common renal malignancy in children -most are solitary and encapsulated (surrounded by pseudocapsule- 5 to 7% bilateral) -s/sx: firm smooth abdominal mass that doesn't cross the midline, pain, hematuria, fever, HTN, and ascites present if capsule ruptures-once suspected abdominal palpation should be limited to prevent capsule rupture causing tumor to spread

Diagnosing Pediatric Cancers

1.Complete history and physical exam with vital signs-child's PMHx -family hx -history of present illness w/ ROS -physical exam in head-to-toe fashion2. Review of Systems:-ask about any fatigue, pallor, pain, recent weight loss-dyspnea-excessive or easy bleeding -HA, N/V -changes in bowel habits-recurrent fevers -behavioral changes, visual disturbances

Treatment Modalities for Pediatric Cancers

Surgery: 1. may include tumor debulking or resection, incisional or excisional biopsy, CVL placement (incl. port-a-cath) 2. Post-operative Nursing Care -admin pain/nausea meds and ABX as ordered -may have epidural (freq. site checks) &/or PCA -assess surgical site- bleeding, s/sx infection -freq. physical assessment, monitor VS closely-assess for s/sx of increased ICP following brain tumor resection -strict I&O's (NGT, emesis, UOP, drains [EVD, JP, Hemovac, etc.], hest tube(s), IVFs, PO intake-follow labs if ordered (CBC, CMP, pathology, etc.)-incentive spirometry -early ambulation when possible, PT consult Radiation: -damages or kills cancer cells -may be used as curative, adjunct, or palliative therapy -shields must be used to protect healthy tissue from damage -site marked by radiologist, important not to remove -A/E: fatigue, N/V, mucositis, myelosuppression, damage to skinChemotherapy: -agents work to alter the cancer cell during a specific point in the cell cycle or act on the cancer at any point during its cell cycle. -alter normal cells as well -> A/E ass.w/ chemo-combo. of drugs given during different phases of the cell cycle to maximize destruction of cancer cellsCommon Side Effects to Cancer Treatment Modalities: -infection, pain, anemia or blood loss-GI sx like N/V, diarrhea, anorexia and constipation-damage to oral mucosa/skin, alopecia

Nursing Management for Chemo and reducing its side effects

-avoid spicy foods or strong odors -give antiemetic 30 min before chemo and regularly as ordered for 24 hours after chemo ends-relaxation techniques -test emesis for blood -hydration and fluids

Nursing Care for Pediatric Cancer Patient

Infection Prevention: -regularly assess for potential infections -admin prophylactic ABX as ordered -promote and enforce proper hand hygiene -promote nutrition and periods of rest -educate caregivers surrounding steps to prevent infection (limit visitors, avoid sick contacts and crowds, encourage vaccination [NOT LIVE VACCINES] once approved by provider, monitor for s/x infection [no rectal temps] and notify oncologist or bring child to ED if febrile-neutropenic precautions if indicated Neutropenic Precautions: -private room, closed door -frequent hand hygiene -avoid rectal temps, enemas, and invasive procedures like catheterization -no fresh fruits/veggies, flowers or plants in room -visitor restriction -VS per protocol -child wears mask if they leave the room Prevention of Anemia and Blood Loss: -assess for s/sx of bleeding (bruising, pallor, lightheadedness, changes in VS) -avoid rectal temps and exam (damage to rectal mucosa -> bleeding) -pressure to potential sites of bleeding (injection sites, bone marrow bx site) -limit blood draws/injections -encourage intake of iron rich foods -PLT and EPO admin as ordered Management of GI symptoms: -admin. antiemetics (before onset of nausea and PRN) and antidiarrheals as needed -hydration therapy (oral/IV); Parenteral nutrition (TPN/lipids) as ordered -strict I&Os -monitor weight frequently-protect perineum from skin breakdown-det. preferred foods, provide high cal meals when they're most hungry, offer supplemental shakes-offer small meals more frequently -constipation-> increase fluid and fiber intake, admin. stool softeners or laxatives as ordered Skin Protection:-avoid adhesives -gentle soaps/cleansers and pat skin dry-avoid fragrant lotions, deodorants, and soaps -moisturize skin frequently -admin Benadryl &/or hydrocortisone cream for itching -apply Silvadene cream to areas of desquamation -avoid direct sun and wear loose-fitting clothes following radiation Protection of Oral Mucosa: -frequent assessments, keep mucosa moist (oral hydration, ice chips if NPO, petroleum jelly to lips) -salt water rinses/mouthwash (w/o alcohol) every 1-2 hr. -soft toothbrush (prevent gum irritation/bleeding) -if mucositis present admin. magic mouthwash as ordered -avoid spicy/acidic foods -pain meds as ordered Promote Mobility and ADL's-encourage ambulation and activity-monitor for s/sx intolerance (dizziness, pallor, lightheadedness, orthostatic hypotension) -cluster nursing care and offer periods of rest -ROM exercises and position changes if limited mobility or on bedrest -PT/OT consult Promote Self-Esteem and Body Image: -explore strengths and weaknesses, provide feedback -encourage to perform ADL's when able -schoolteacher consult -offer self-care activities and emotional support -encourage participation in support groups-allow child to make decisions about appearance and spend time w/ peers who have or are undergoing similar experiences -discuss counseling referral w/ child, caregivers, providersPromote Coping Mechanisms and Sense of Control: -consult child life specialist (CLS) prior to invasive exams or procedures -educate and allow child to make decisions about care when appropriate-encourage child to spend time w/ friends, support/teen groups-open communication, allow child to express emotions, acknowledge feelingsDeliver Patient and Family-Centered Care: -provide emotional support -education surrounding infection prevention measures, management of pain and other treatment side effects, preparing for home health needs (med admin, CVL care, equipment needs, etc.)-CLS consult for siblings, chaplaincy consult -inform about community resources (support groups, grief counseling) -caregiver burnout

Hematopoietic Stem Cell Transplant (HSCT)

Two main types: (Autologous): uses the child's own stem cells; stem cells are removed and frozen, then chemotherapy or radiation occurs, finally cells are thawed and reinfused. (Allogenic): uses a matched donor's stem cells, HLA matchingExtensive Evaluation and Preparation -evaluation from transplant team -family planning (prolonged hospitalization) -CVL placement -HLA matching (allo) -stem cell harvest (also or auto) -myeloablative therapy Pre-transplant Nursing Care: -infection prevention -isolation in neg. pressure room -limit visitors (caregivers) -admin prophylactic ABX -antifungals -antivirals (acyclovir, flu shot) -frequent hand hygiene -provide CVL site care -regular CHG bathing-admin blood products PRN -admin granulocyte colony stimulating factor (G-CSF) if ordered -provide child and family education and supportPost-transplant Nursing Care: -prevent and monitor for graft-versus-host disease (GVHD) a) admin prophylactics (cyclosporine and methotrexate) b) initial s/sx: maculopapular rash -> skin desquamation, diarrhea c) may require systemic steroids if GVHD occurs -manage GI sx-infection prevention -bleeding and pancytopenia (blood products PRN)-admin G-CSF as ordered for autologous HSCT recipients (Filgrastim- IV or injection)

Calculation of Absolute Neutrophil Count (ANC)

<1000 = neutropenia WBC x total neutrophils (%segs + % bands) x10Ex: 2.5 (20 + 10) = 750

Oncologic Emergencies

Superior Vena Cava (SVC) Syndrome: mediastinal mass (NHL or neuroblastoma) -> compression of SVC -s/sx: dyspnea, cyanosis, wheezing, diminished breath sounds -management: intubation and ventilation, surgical intervention, comfort measuresNeutropenia Enterocolitis (Typhlitis): life-threatening, necrotizing enterocolitis, occurs mainly in pts w/ blood cancers (ALL) following induction of chemo -> disruption of integrity of intestinal mucosa -> opportunistic pathogens cause infection and necrosis of intestinal wall -s/sx: abdominal pain, N/V, fever, anorexia, bloody emesis or diarrhea -management: ABX therapy, admin pain meds, NPO/TPN, monitor for s/sx of perforation, comfort measures Increased ICP: Brain tumor or brain metastasis compressing the brain -> herniation -s/sx: HA, vomiting, altered LOC, irritability, seizures, Cushing's triad-management: steroids, anticonvulsants, radiation, tumor resection, comfort measures Sepsis: life-threatening organ dysfunction caused by dysregulated pt. response to infection -s/sx: tachycardia/pnea, fever or hypothermia, respiratory distress, poor perfusion, altered LOC, pallor, sx specific to infectious source (ex: cough w/ PNA, flank pain w/ pyelonephritis)-management: maintain airway and ventilation, fluid volume resuscitation, inotropic support, admin. broad spectrum abxTumor Lysis Syndrome

Tumor Lysis Syndrome (TLS)

Pathogenesis: 1. initiation of therapy (chemo, radiation, glucocorticoid) -> rapid lysis of tumor cell -> intracellular contents (K+, PO4, uric acid) released into systemic circulation -> metabolic consequences 2. metabolic consequences include hyperkalemia, hyperphosphatemia, secondary hypocalcemia, hyperuricemia, AKIClinical Manifestations: -N/V, diarrhea, anorexia -lethargy, muscle cramps, dysrhythmias, heart failure -seizures, tetany, syncope, hematuria Prevention: -admin hypouricemic agents (Allopurinol) prophylactically and during treatment -IVFs at 2x maintenance -freq. monitoring serum electrolytes, urine output, and serum uric acid Management: 1. Correct electrolytes-hyperkalemia -> limit K+ intake, cardiac monitoring, freq. serum K+ levels, admin glucose w/ insulin or calcium gluconate as ordered to decrease risk for dysrhythmia -hypocalcemia -> if symptomatic treat w/ the lowest dose of Ca -hyperphosphatemia -> aggressive hydration, phosphate binders 2. Renal replacement therapy, hemodialysis

Anatomy and Physiology Differences in Pediatric GI System

Upper Gi Tract: -lower esophageal sphincter is fully developed at 1 mo. -stomach capacity in infancy is ~200 mL (adult 2,000-3,000 mL) so gastric emptying faster than in adults Lowe GI Tract: -small intestine ~250 cm vs 600 cm in adults -healthy infants have faster intestinal transit time than adults -liver is ~1/2 size of abdominal cavity, edge may be palpable on exam 2-4 cm below the right costal margin -immature neuromuscular function, often stool after eating as a result of the gastrocolic reflux by ~2 years voluntary control of BMs occurs Meconium- first stool and appears black in color, sticky

Gastroesophageal Reflux Disorder (GERD)

Physiologic Gastroesophageal Reflux: passage of gastric contents into the esophagus (regurgitation or spitting up) -frequent in infants < 12 months -physiologic in all ages until it causes sx of esophageal injury or complications -> referred to as GERD GERD: -s/sx: may be vague and include poor weight gain (FTT), feeding refusal, irritability, respiratory sx (cough, wheezing), neck or back arching, heme + stools -underlying causes must be ruled out prior to dx (ex-food allergy, malrotation) -conditions ass.w/ GERD -> obesity, CF, neurologic impairment, prematurity, hiatal hernia, achalasia) Nursing Care: -Administer feedings 1. Elevate head or HOB during feedings 2. keep infant upright for at least 30 min. after feeding 3. smaller, more freq. feedings; avoid overfeeding 4. Thicken feeds (oat cereal) -other care:1. GERD should resolve spontaneously as infant grows 2. moderate-severe cases infants may require meds (PPI- Omeprazole, H2 receptor antagonist- famotidine) 3. Nissen fundoplication reserved for infants w/ severe GERD, refractory to above treatment

GERD pediatrics

Transfer of gastric contents into esophagus causing: -regurgitation -poor growth -choking -irritability -blood in stoolNursing management: -supine and upright positioning -thickened foods

Cleft Lip and Palate

Etiology: -most common congenital craniofacial abnormality, ~30% ass.w/ genetic syndromes -infants at higher risk to develop if mother smokes, takes certain medications, or is old Clinical Manifestations:-many present w/ both cleft lip and palate (~50%); may present w/ cleft lip alone or cleft palate alone; may be left, right sided or bilateral-cleft lip can be visualized on exam-cleft palate can be visualized on exam and palpated w/ gloved finger -lip is repaired at 3 mo. and palate at 6 mo.*difficualty feeding, frequent ear infections, speech difficulty, aspiration Nursing Care: -Promote nutrition (cleft palate alone) -> infant may not be able to generate enough pressure to suck milk from bottle or breast. Increased risk for aspiration w/ cleft palate, covering of palate w/ prosthodontics device should be used for prevention. Adaptive equipment may be used to assist in feedings (squeezable bottle or modified nipple) (cleft lip alone) -> infant can feed from bottle or breast -Encourage Parent-Infant Bonding -distress surrounding appearance and hospitalization -support parent in providing feedings and other care -provide education for anticipated surgeries Post-operative Care: -pain management w/ medications and comfort measures (holding, rocking, singing), anticipate needs -prevent damage to suture lines of lip and/or palate (infant should lay supine, may require mitten or welcome-sleeve restraints) -don't allow infant to suck on pacifier, plastic syringe, or straws in older children undergoing repair-no oral suctioning***maintain airway, arm restraints, nutrition, positioning, protect sutures

What is the best position for an infant who just had a cleft lip repair?

Supine

Hypertrophic Pyloric Stenosis

Narrowing of the pyloric sphincter that blocks passage of food from stomach to small intestine (olive shaped mass in RUQ)- vomiting, metabolic alkalosis, increased BUNEtiology: -hypertrophy of the pylorus -> obstruction of gastric outlet -> forceful, often projectile vomiting Clinical Manifestations: -forceful, often projectile vomiting -occurs early in infancy often between ~3 to 6 weeks -vomiting occurs immediately after feeding -infants irritable and hungry, wanting to feed soon again after emesis -s/sx:failure to gain weight or weight loss, dehydration (dry mucous membranes, no tears), hypochloremic metabolic alkalosis, palpation of olive-sized mass in RUQ -DX: abdominal US will demonstrate hypertrophied pylorus Surgery Treatment: Laparoscopic PyloromyotomyNursing Care (preoperative): -admin IVFs to correct dehydration and electrolyte disturbances -bolus of NS-MIVFs at 1.5 to 2x maintenance rate-NPO before surgery -edu and support to caregivers surrounding surgery Nursing Care (postoperative): -resume oral feeds and continue IVFs until tolerating PO -PRN pain meds -strict I&Os-monitor surgical site

intussusception

Telescoping of part of the intestines into an adjacent part -> sausage shaped mass, jelly-like stools, crying w/ knees to chest, vomiting. Management: prepare for barium enema, observe for sepsis/shock, monitor stoolsMost common cause of intestinal obstruction in children 6 months to 3 years of age.Bowel telescopes into a distal portion of itself, if left untreated -> edema, vascular compromise, and bowel ischemia.-classified by location (ileocolic ~90%) Clinical Manifestations: -may be preceded by viral illness-most common causes are idiopathic, some may have a lead point (~25%)-s/sx: sudden crampy intermittent abdominal pain; severe abdominal pain , child may draw knees to chest, vomiting, diarrhea, lethargy, currant-jelly stools*, sausage-shaped mass on abdominal palpation Nursing Care: (Stable Child)-> admin IVFs. Transport to radiology for air (pneumatic) or contrast (hydrostatic) enema. Educate caregivers surrounding procedure and likelihood of recurrence (~50% reoccur in first 72 hours following nonoperative reduction) (Unstable child w/ s/sx bowel perforation or unsuccessful nonoperative intervention) -> emergent surgical intervention, admin IVFs and ABX, NGT decompression, post-operative care- important to assess for s/sx recurrence, caregiver support and education surrounding s/sx of recurrence

Hirschsprung Disease

Absence of ganglion cells in colon causes problems passing stools.Neural crest cells fail to migrate completely during intestinal development -> aganglionic segment of colon -> lack of propulsion of intestinal contents (functional obstruction)-aganglionic segment may be short (rectosigmoid), long (extends proximal to sigmoid colon) and total colonic aganglionosis -outcomes worse w/ long-segment disease -ass.w/ chromosomal anomalies (trisomy 21) *hallmark is delay or failure to pass meconium in first 24 hours of life Clinical Manifestations: -s/sx: abdominal distention, poor feeding, constipation, bilious emesis, squirt or blast signHirschspring-Associated Enterocolitis (HAEC): bacterial overgrowth in bowel lumen from stool stasis, translocation of bacteria through mucosa -s/sx: fever, vomiting, lethargy, abdominal pain, explosive and foul-smelling diarrhea-may occur pre or postoperatively (most common) -can lead to intestinal perforation, peritonitis, septic shock, and deathTreatment: -surgical correction required to resect affected segment of the rectum and colon, bring the normal ganglionic bowel down to an anastomosis to the distastes rectum, close to the anus-may be laparoscopic pull-through or open (laparotomy) pull-through; may require multiple surgeries Pre-operative Nursing Care: -NPO and gastric decompression w/ NGT-IVFs and IV ABX -Rectal irrigationsmonitor for s/sx of HAEC-caregiver support and education Post-operative Nursing Care: -pain management -NGT decompression until return of bowel function (ROBF) -monitor for s/sx HAEC (notify provider, IV ABX and irrigations will need to be started)-surgical site care and stony assessment/site care if present -caregiver support and educationRecommend high fiber diet

Imperforate Anus

Anal tract fails to develop Normal anal opening is absent, colon empties onto the perineum, or toward the vagina (female pts) or into the urinary system (male). Clinical Manifestations: Atresia may be present->rectum is blind ending w/ no connection or external openingAssociated Anomalies: All newborns with imperforate anus need complete VACTERL workup. 1. Vertebral- tethered cord, scoliosis, myelomeningocele 2. Anal- atresia 3. Cardiac- ASD, PDA, tetralogy of Fallot 4. GI- transesophageal fistula 5. Renal- vasicoureteral reflux 6. Limb abnormalitiesNursing Care: (Nonoperative Patient- fistula present) -> prepare and transport for dx tests. fistula dilations as ordered. caregiver support and education. will need eventual surgical repair (~ 3 mo.), dilations at home, s/sx of inadequate dilations. typical newborn care as well. (Infant w/o fistula) -> 1st surgery colostomy to divert stool and gas; 2nd surgery posterior sagittal anorectoplasty; final surgery colostomy reversal. -pre-op-> abx, limited clear or breast milk diet 24 hours before surgery, continue dilations-post-op-> routine post care for all procedures (pain management, advance diet as tolerated, monitor surgical site); stony care (1st and 2nd), bacitracin to surgical site and nothing per rectum (final)

Omphalocele

Herniation of abdominal contents into umbilical cordFull thickness and midline abdominal wall defect that allows evisceration of the abdominal contents into an external peritoneal sac; umbilical cord inserts into the omphalocele membrane, not the abdominal wall; pulmonary hypoplasia may be present.-Ass.w/ advanced maternal age and infants w/ chromosomal anomalies -defects usually larger than gastroschisisNursing Care: -cover sac w/ sterile gauze or bowel bag up to infant's chest; careful to avoid rupture of sac-position LEFT side down right side up if vascular compromise suspected (hypotension, tachycardia, or dusky bowel appearance)-insert OGT and place to suction, IVFs & ABX, strict I&O-'paint and wait' technique for large defect or unstable infant **application of bacitracin ointments to the sac to allow for eschar then epithelialization to develop over time -> eventual ventral hernia which can be repaired later **prepare for silo placement (staged closure) or operative intervention (primary repair) -postop pain management, OGT to suction until ROBF, advance feedings once bowel function returns, strict I&Os, monitor for s/sx of obstruction

Gastroschisis

Herniation of the bowel through abdominal wall defect to the right of the umbilicus.Herniation of the abdominal contents through the ventral abdominal wall defect, at the right of the umbilicus, umbilical insertion to left of defect-not enclosed w/in a sac, exposure to amniotic fluid -> thickening of herniated organs and peel may be present-high risk for developing malrotation, intestinal obstruction -increased risk in infants of young mothers (<20) and maternal obesity Nursing Care: -cover exposed viscera w/ bowel bag to maintain temp and fluid losses, observe bowel for decreased perfusion (dusky, cool), position RIGHT-side down to decrease risk of vascular compromise (removes tension from mesentery) -insert OGT & place to suction -resuscitation w/ IVF then MIVFs at regular rate, admin ABX, strict I&Os-prepare for silo placement (in staged closure) or operative intervention (primary repair) -postoperative-> pain management, OGT to suction until ROBF, advance feedings once bowel function returns, strict I&Os, monitor for s/sx of abdominal obstruction syndrome

Omphaalocele and Gastroschisis pre-op nursing care

-prevent heat loss (hypothermia common) -cover abdominal contents with WARM, sterile dressing -place on side -NG tube for decompression

Currant jelly stools are a hallmark diagnosis of which disease?

Intussusception

Anatomy and Physiology Differences in Pediatric GU System

-kidneys large in abdomen, at increased risk for trauma -shorter urethra in both sexes during infancy-slower glomerular filtration rate, kidneys less efficient at regulating F&E and acid/base balance -smaller bladder capacity (20-50 mL in infants vs adults 700 mL) -urine output should be ~ 2 mL/kg/hr in infants, between 0.5-1 mL/kg/hr in older children -renal system reaches maturity at 2 years

Urine Collection in Pediatric Patients

-discuss catheterization techniques w/ parents and obtain permission -involve CLS for older children before invasive collection -urine bag (appropriate for infants and toddlers; may be sterile for urine culture or clean for urinalysis) -Straight Catheterization (for UA or culture, in cases of retention or obstruction, technique is the same as in adult pt. but may be difficult due to smaller anatomy and pt. cooperation)

Urinary Tract Infection (UTI)

Etiology: -infection of the urinary tract, most commonly the bladder from bacterial transmission via the urethra-females at increased risk d/t shorter urethra length and proximity of urethra to vagina and anusClinical Manifestations: (infants)-> irritability, fever, poor feeding, vomiting, jaundice (children)-> fever, vomiting, dysuria, incontinence, urinary hesitancy and urgency, hematuria, abdominal pain, flank pain, foul-smelling urine (if left untreated)-> pyelonephritis, urosepsis, renal scarring, hydronephrosis Diagnostics: -UA w/ reflux to culture -positive for blood, nitrites, leukocyte esterase, WBCs, or bacteria; culture will reveal infecting organism Nursing Care: (home care) -ensure adequate hydration -avoid urinary retention, frequent bathroom breaks (note for school-aged children) -females no bubble baths or soap application to vulva, wipe front to back, cotton underwear, change pads/tampons frequently, void after intercourse -complete full ABX course and return to PCP for re-check -pain management and antipyretics (acetaminophen, ibuprofen, small children may void in warm bath, heating pad) (hospital care) -criteria: infant <2mo., s/sx urosepsis, immunocompromised pt., inability to tolerate oral meds -care: admin ABX therapy, hydrate via oral or IV fluids, pain management and antipyretics PRN

Infants/Toddlers non-classic sx of UTI

fever, poor feeding, vomiting

Hypospadias/Epispadias

Abnormal location of the urethral opening*Hypospadias- congenital anomaly of the male urethra, foreskin, and penis leading to abnormal ventral placement of the urethral opening*Epispadias- defect in which the urethral openings on the dorsal surface of the penis Abnormal displacement has potential to cause developmental or functional issues:-deflection of urinary stream -inability to urinate standing -ED causing intercourse difficulties -problems w/ fertility given altered deposition of spermTreatment (Surgery): -circumcision contraindicated before surgical repair-surgical repair occurs around 6 to 12 months of age and is usually outpatientPost-operative Nursing Care: caregiver support and education surrounding ->-pain med admin (analgesics like acetaminophen and ibuprofen) and antispasmodics l(ike oxybutynin)-ABX given postoperative to prevent infection while catheter is in place -surgical site care (dressing stays in place ~3 days, soak off, may apply Vaseline to outer diaper once dressing comes off) -double diapering and catheter care -when to call provider or go to ED (fever, excessive bleeding, inconsolable pain, change in urine output)

Vesicoureteral Reflex

Etiology: -retrograde passage of urine from the bladder into the upper urinary tract -> renal scarring; potential for renal insufficiency or failure later in life*Primary-> congenital abnormality, inadequate closure of uterovesical junction -> failure of anti-reflex mechanism *Secondary-> result of high pressure in the bladder -> failure of uterovesical junction closure; may be caused by neurogenic bladder, obstruction or dysfunction of the bladder Diagnostics: -dx by voiding cystourethrogram (VCUG) -severity graded on I-V scale -Grades III-V ass.w/ recurrent UTIs, hydronephrosis, and renal injury Nursing Care: -daily admin of prophylactic ABX until VUR is resolved (spontaneously or surgically) -Edu child and caregiver around proper toileting & perineal hygiene, need for regular urine testing and VCUGsPost-operative Nursing Care: -IVFs 1.5 maintenance rate to promote urinary output -strict I&Os-foley and stents may be in place; urine will be bloody for 2-3 days postoperative -admin analgesics and antispasmodics -ambulate, advance diet as tolerated

Nephrotic Syndrome

Etiology: -increased permeability across the glomerular filtration barrier -> passage of plasma proteins through basement membrane -> loss of protein in urine and decreased protein and albumin in serum-Hypoabluminemia -> change in osmotic pressure -> fluid shifts into interstitial tissue, shift in volume prompts kidneys to conserve water and Na -> edema -may be primary or secondary *Primary-> nephrotic syndrome in the absence of diseases, includes idiopathic which is most common form *Secondary-> nephrotic syndrome ass.w/ systemic diseases or is secondary to another process that causes glomerular injury (ex- Systemic Lupus Erythematosus, Henoch-Schonlein Purpura, Sickle Cell Disease) Clinical Manifestations: -proteinuria -hypoalbuminemia -edema (generalized- anasarca) -hyperlipidemia Nursing Care: -Promote diuresis -admin corticosteroids (prednisone) & -admin diuretics (furosemide) as ordered -daily weights -monitorUOP and proteinuria -regular assessment of VS and edema -admin albumin for severe hypoalbuminemia-Infection prevention -give prophylactic ABX as ordered -vaccine admin (pneumococcal, varicella) *avoid live vaccines during steroid treatment -Nutrition -may req. fluid and Na restriction -encourage foods high in protein and K+ *K+ if low from diuretics -Child and Caregiver Support -surrounding course of treatment & -preventing relapse by adhering to tx plan -teach how to check proteinuria via dipstick -offer supportive resources if child distressed w/ appearance

Post-streptococcal glomerulonephritis

-Immune complex disease that occurs after an infection-infection -> immunologic response, immune complexes deposit into glomerular membrane -> glomerular inflammation and injury, can lead to uremia and renal failureClinical Manifestations: -recent infection, fatigue, hematuria, proteinuria, decreased glomerular filtration rate, and retention of Na and water -> edema and HTNNursing Care (supportive): -ABX therapy if strep infection present -monitor and manage HTN w/ antihypertensives and diuretics -daily weights -Na and water restriction -strict I&Os -cluster care to allow for rest Treatment: -may require dialysis for life-threatening fluid overload refractory to treatment, uremia, elevated K+ resistant to treatment

Hemolytic Uremic Syndrome (HUS)

Etiology: -3 defining features: 1. hemolytic anemia 2. thrombocytopenia 3. acute renal failure *Acquired HUS-> 1. Shiga toxin producing E. coli (STEC 0157:H7 most frequent in US) accounts for 90% of cases of pediatric HUS; E. coli 0157 transmitted via undercooked beef, feces of certain animals, unpasteurized dairy; s/sx: abdominal pain, vomiting, diarrhea (usually bloody); HUS s/sx begin 5-10 days after diarrhea onset 2. pt. may progress to severe AKI or renal failure, ~ 50% of patients will need dialysis 3. neurologic s/sx may be seen- altered mental status, seizures, coma, stroke Nursing Care: 1. Anemia -slow infusion of PRBCs as ordered (3-4 hours) 2. Thrombocytopenia -PLT transfusion only if active or severe bleeding is present 3. Maintain fluid balance -if decreased intravascular volume- admin fluids to restore euvolemic state -if increased intravascular volume-admin antidiuretic and antihypertensives as ordered 3. Kidney monitoring and protection -labs: CBC, creatinine, BUN, electrolytes-avoid admin of nephrotoxic meds -manage HTN- restore euvolemia, admin diuretics or antihypertensives as ordered -pt may require dialysis -education should be given surrounding risk of transmission (STEC is shed for ~ 7 days, longer in younger pt ~3 weeks)

Respiratory Disorders (Alterations in Gas Exchange)

most common cause of illness and hospitalizations in children. several factors play into this: -child's age (smaller anatomy) -socioeconomic status-general health status -season (winter and fall) Child's Age: -obligate nose breather until at least 4 weeks old (if nose is occluded, they don't automatically open their mouth-infant trachea 4mm wide (2 and 1/2 times as wide as a strand of spaghetti) -adult trachea is 20 mm (size of penny)-fewer alveoli than adults (smaller area for gas exchange puts child at risk for hypoxia)-infant chest walls are VERY FLEXIBLE and do not support lungs like firmer adult ribs and sternum-tidal volume is dependent on movement of diaphragm; if it can't move appropriately, intercostal muscles cannot lift chest wall appropriately; DIAPHRAGMATIC BREATHERS

Auscultate Lung Sounds

-if child comfortable in caregivers arms, leave them there-warm stethoscope Assess for: *Wheezing- high pitched usually on expiration (asthma, bronchiolitis)*Rales- crackling sound (pneumonia) *Stridor- heard on inspiration; croup, upper airway obstruction

Pediatric Respiratory Nursing Care Management

Ease respiratory efforts:-warm or cool mist -oxygen -saline nasal spray Promote comfort: -positioning (babies slip down when they sleep so make sure to keep the airway open)

Common Tests for Children with Respiratory Disorders

-allergy skin testing (pt must be off antihistamines) -peak expiratory flow (common w/ asthma) -RAST; radioallergosorbent test (food allergy test, not super accurate) -sweat chloride test (gold standard for cystic fibrosis; how much salt is in the sweat)

Acute Nasopharyngitis (common cold)

-upper respiratory infection -fever is common in young and older children -therapeutic management: Rest, cool mist humidifier, fluids, elevate HOB, saline to nose -potential complications: bacterial infections of ears, throat, sinuses, or lungs(happens bc congestion just sits there)

Acute Otitis Media

Infants and children more susceptible due to shorter and more horizontal Eustachian tubes -bottle propping and second-hand smoke causes it to occur more often-Prevention: breastfeeding infants if possible; avoid second hand smoke

Influenza

Therapeutic management is symptomatic! -antipyretic medication -antiviral influenza meds (when appropriate) -avoid Tamiflu

Acute Infectious Pharyngitis

CM: pharyngitis, HA, fever, and abdominal painGroup A strep throat: sandpaper rash (scarlatina)Complications of Strep Throat:-rheumatic fever (leads to rheumatic heart dx) -acute glomerulonephritis onset (painless hematuria)-peritonsillar or retropharyngeal abscess (hot potato/ muffled voice)

Croup

Infections of epiglottis and larynx (upper airway) causing hoarseness, brassy cough, or inspiratory stridor. *WHAT PROFESSOR SAID TO MEMORIZE:Barky seal cough (stridor) Symptoms most often at night (lasts 3-5 days)Therapeutic Management: -cool mist vaporizer-steroids -nebulized racemic epinephrine (if stridor at rest)

Epiglottitis

Inflammation of the epiglottis caused by bacterial infection, potentially life-threatening, sx include: -severe soar throat -drooling -leaning forward -dysphagia/dysphonia -respiratory distress -stridorTX: IV abx, keep tract at bedside*WHAT PROFESSOR SAID TO MEMORIZE:MEDICAL EMERGENCY -cause: H. influenzae -prepare for intubation; have emergency equipment at bedside -prevention: Hib vaccine

Bronchiolitis

Acute lower airway infection commonly caused by RSV (Respiratory Syncytial Virus) that results in :-increased mucus secretion -fever -tachypnea -wheezing -nasal congestionTreatment: -nasal suctioning -increase fluids -possibly bronchodilators or steroids -Synargis (antibody injection) for high risk*WHAT PROFESSOR SAID TO KNOW:Clinical Manifestations: -initial -> rhinorrhea (runny nose), pharyngitis, coughing, wheezing, low-grade fever-progression of illness -> cyanosis (severe respiratory distress), apneaTherapeutic management: humidified oxygen and fluids, contact isolation

Pneumonia

Cause: -infant/young child -> viral, bacterial (streptococcus pneumonia)-school aged children/ adolescents -> mycoplasma, fungal, or aspiration

Tuberculosis

Hallmark symptoms: weight loss and night sweats

Asthma

Cause: chronic inflammation of airways w/ char. of: -airway hyper responsiveness (bronchoconstriction) -airway edema -mucus production increased 3 A's: -asthma -allergies -atopic dermatitis (eczema) Asthma Action Plan: -all kids should have it: stoplight -stepwise approach w/ 3 different levels -adds to treatment as you goMedication Management: -corticosteroids (inhaled and oral) -B-adrenergic agonists (SABAs and LABAs) -mast cell stabilizers -leukotriene modifiers: montelukast (Singulair)-> have behavioral changes; aggressive in kids-anticholinergics

Cystic Fibrosis

Genetic disorder of exocrine glands resulting in viscous mucous production that affects the lungs and pancreas -> meconium ileum, frothy fat-containing stools, chronic respiratory infections, and malabsorption.*what professor said to memorize:Cause: inherited autosomal recessive gene from both parents. Effects more than the lungs! Pancreatic enzymes are lost. Pathophysiology: increased viscosity of secretions from sweat glands, GI tract, pancreas, respiratory tract, and other exocrine tissues Clinical Manifestations: pancreatic enzyme activity is lost, malabsorption of nutrients, failure to thrive Diagnostic Evaluation: there is newborn screening but the sweat chloride test is the gold standard and only done to confirm the dx if cystic fibrosis is suspected GI management: pancreatic enzyme replacement to help breakdown fats- pancrelipase (Pancrease, Creon)

Circulatory Changes from Gestation to Birth

-triggered by the first breath -PA pressure drops ->causes closure of the ductus arteriosus -RA pressure drops -> causes the closure of the foramen ovale -vasoconstriction -> causes closure of the ductus venosus

Cardiovascular Status

Factors Influencing: -genetic (influence) -environmental -maternal -multifactorial Types of Developmental and Biologic Variances in Cardiovascular Status: 1. those that occur during embryologic development 2. those that manifest as a result of cardiovascular changes at birth 3. those that occur during childhood development

Cardiovascular Nursing Assessment (pediatric)

Health History: -Patient's hx (birth hx, infections, chromosomal anomalies, prematurity, autoimmune dx, meds)-maternal and fetal hx -postnatal hx -family hx Inspection: -cyanosis, SOB, difficulty breathing (DIB), tachypnea, clubbing, eating difficulties, failure to thrive (FTT) -activity and general appearance -edema or jaundice Palpation: -apical impulse (AI) -peripheral pulses and capillary refill time (CRT) -liver bordersAuscultation: -heart sounds -rate and rhythm -compare upper and lower extremities BP

Making a Diagnosis

-depends on the sx -Holter monitor -Chest X-Ray (CXR) -Electrocardiogram (ECG) -Echocardiogram -Arteriogram -Prenatal Ultrasound (US) -Cardiac catheterization

Catheterization Procedure

Pre-catheterization Assessment: -obtain thorough health hx and physical exam -est. baseline for post-cath -obtain baseline VS -note fever or other s/sx of infection -note allergies -review meds -note the NPO status -review labs Postcatheterization Assessment: -record VS frequently -monitor for hypotension and bleeding -check insertion site -monitor and compare catheterized extremities -assess the child's LOC-BP on all 4 extremities

Congenital Heart Disease

Decreased pulmonary blood flow: -Tetralogy of Fallot -Tricuspid Atresia Increased pulmonary blood flow: -ASD (Atrial Septal Defect)-VSD (Ventricular Septal Defect)-AV Canal -PDA (Patent Ductus Arteriosus)Obstructive: -Coarctation of the Aorta -Aortic Stenosis -Pulmonary Stenosis Mixed: -Transposition of the great vessels -Total Anomalous Pulmonary Vein Connection -Truncus Arteriosus -Hypoplastic Left Heart SyndromeTetralogy of Fallot: -results in decreased pulmonary blood flow-4 defects: 1. right ventricular hypertrophy 2. overriding aorta 3. pulmonary stenosis 4. VSD (ventricular septal defect)-hypercyanotic spells or "Tet Spells" treatment -> use a calm, comforting approach; place in a knee-to-chest position; provide supplemental O2; admin morphine (IV, IM, or SQ); supply IV fluids (tet spells- resulting from decreased pulmonary blood flow in Tetralogy of Fallot-> hyper cyanotic spells)Tricuspid Atresia: * absence of tricuspid valve, blood doesn't go into the right ventricle and deoxygenated blood passes through the atrial septum into the left atrium resulting in decreased pulmonary blood flow.-decreased pulmonary blood flow -tricuspid valve does not develop in utero -blood does not go directly into the right ventricle -deoxygenated blood passes through the atrial septum (PFO) -blood mixing at Pulmonary Artery and AortaAtrial Septal Defect (ASD):*a hole in the wall separating the atria causes increased pulmonary blood flow, SOB, ad FTT over time -increased pulmonary blood flow -hole in the wall dividing the left and right atria -often asymptomatic -increased blood flow results in SOB, fatigue, and failure to thrive (FTT) over time Ventricular Septal Defect (VSD):*hole in the septum between the ventricles that results in increased pulmonary blood flow, a loud harsh murmur, blood shunting to the right side of the heart, pulmonary hypertension, and heart failure if not repaired -increased -hole in the wall between the left and the right ventricles -asymptomatic if small -left to right shunt -loud, harsh murmur -increased flow to lungs leading to PHTN -heart failure if not repaired -FTT Atrioventricular Canal (AV Canal): *incomplete fusion of the endocardial cushions resulting in ASD and VSD that allows blood to flow between all 4 chambers -> blood shunts from left to right resulting in increased pulmonary blood flow and pulmonary edema-increased -failure of endocardial cushions to fuse -tricuspid and mitral valves do not get separated -ASD and VSD are present -left to right shunting -pulmonary edemaPatent Ductus Arteriosus: *passageway between the aorta and the pulmonary artery that results in increased pulmonary blood flow-increased -second most common congenital heart defect -more common in premature infants and those born in high altitudes -can occur to accommodate right to left shunting diseases -persistent connection btwn aorta and pulmonary arteryCoarctation of the Aorta (Coarct): *Narrowing of the lumen of the aorta the obstructs blood flow -> BP in all 4 extremities, upper extremities will have higher BP than lower-obstructive disorder -narrowing of the lumen of the aorta -BP in all 4 extremities (upper will be higher than lower) Aortic Stenosis: *restricted blood flow from left ventricle into the aorta resulting in obstruction, FTT, faint pulses, and chest pain. -obstructive disorder -restricted blood flow from left ventricle to aorta -typically asymptomatic (FTT, faint pulses, easy fatigue, chest pain) Pulmonary Stenosis: *restricted blood flow from the right ventricle to the pulmonary artery, resulting in obstruction, FTT, faint pulses, and chest pain.-obstructive -restricted blood flow from right ventricle to pulmonary artery -typically, asymptomatic (FTT, faint pulses, easy fatigue, chest pain) Transposition of the Great Vessels: *Congenital abnormality where the aorta is attached to the right ventricle and the pulmonary artery to the left ventricle (backwards from what it's supposed to be)-> mixed defect that requires surgery-mixed defect -PA and aorta are switched -may also have ASD or VSD -treatment: surgical switch, balloon atrial septostomy, PGE Total Anomalous Pulmonary Vein Connection: *mixed defect where the pulmonary veins do not connect to left atrium and instead connect to right atrium or vena cava-mixed defect -pulmonary veins do not connect to the left atrium-instead they connect to the right atrium or SVC-PFO or ASD usually present Truncus Arteriosus: *mixed defect arising from failure of septum formation, resulting in a single vessel that comes off of the ventricles (aorta and pulmonary artery are one) -> decreased systemic blood flow -> requires surgery-mixed defect -single, large vessel -decreased systemic blood flow -requires surgical interventionHypoplastic Left Heart Syndrome: *mixed defect resulting in underdeveloped left side of the heart; small left ventricle, small aortic arch -> requires several surgeries and often transplant.-mixed defect -small left ventricle -small aortic arch -often discovered prenatally-requires several surgeries -often leads to transplant Nursing Management of CHD: -medications as prescribed -improve oxygenation (frequent assessments, Semi-Fowler's, use O2 sparingly) -weigh often -allow for activity and rest -adequate nutrition (increased nutritional needs, oral w/ supplements enterally as needed, high cal feedings, cautious breastfeeding and bottle feeding) -family coping/education -infection prevention

Congestive Heart Disease nursing management

-Semi-Fowler's position -use oxygen sparingly -weigh often -allow for activity and rest -increased nutritional needs, high calorie -infection prevention

What causes the closing of the ductus arteriosus?

PA pressure drops

What causes the closure of the foramen ovale?

Right atrium (RA) pressure drops

Acquired Heart Disease

Heart Failure: -most commonly seen in CHD -most cases occur by 6 mo. of age -cannot pump blood effectively, reduced CO -Hypertrophy -s/sx: sweating during feeds, poor feeding, increased WOB, decreased UOP, poor CO (low BP, tachycardia, and mottled, pale skin), fluid overload (edema and crackles in lungs)Infective Endocarditis: *inflammation of the endocardium affecting the heart valves, resulting in flu-like sx, fatigue, and anorexia-pt. at risk: w/ CHD (congenital heart disease), prosthetic valves, and central lines -causes: bacteria or fungi gain access to epithelium; can carry to other parts of the body -sx: vague flu-like, fatigue, anorexia/weight loss -treatment: abx of anti fungal- long course (4-6 wks) Cardiomyopathy:*disease of the heart muscle causing the heart to no contract properly-myocardium cannot contract properly -most commonly idiopathic -may result in heart failure -often requires transplant -nursing management: !monitor for clots!, vasoactive meds, diuretics, allow some activity but promote rest!Hypertension: -increased prevalence primary HTN-> overweight or obesity ->95th % for gender, age, weight -assessment: groth retardation, obesity -treatment: weight reduction, diet changes, increased activity, pharm treatment Heart Transplant: -over 500 children receive transplants per year -evaluation of candidacy -recovery varies -lifelong tx: anti rejection meds

Signs of Cardiac Tamponade

!! Sudden cessation of chest tube drainage!! -muffled heart sounds -hypotension -narrowing pulse pressures -widening mediastinum on chest X-ray

Treatment Options for Cardiac Disease in Children

- Interventional catheterization - Cardiac Surgery - Pacemakers - Cardiac transplantation - Medical management interventions (Pharmaceutical, Dietary, Activity) - Supportive Care - Community CarePostoperative Interventions: -manage temperature -maiintain cardiac output -prevent cardiac tamponade and arrhythmias -manage temporary pacing wires -maintain F&E balance -promote respiratory function -prevent hemorrhage -monitor neurologic functioning -prevent infection -incision care -manage sedation and pain -manage nutrition -provide psychosocial support S/SX Cardiac Tamponade: -sudden cessation of chest tube drainage -hypotension -muffled heart sounds -decreased systemic perfusion -narrowing pulse pressures -widening mediastinum on chest x-ray -increased right and left atrial pressures Goal of Interventions for a Child w/ Heart Failure: -conserve energy -decrease metabolic demands on the weakened myocardium -remove excessive accumulated fluids -treat coexisting illnesses -maintain normal blood values -positioning -nutrition - !! admin O2 sparingly/cautiously -> pulmonary vasodilator !!Pharmacologic Interventions for Heart Failure: 1. Diuretics- decrease preload -furosemide (Lasix) and spironolactone (Aldactone) -monitor labs and I/O's closely 2. Positive Inotropic Agents- contractility -digoxin (Lanoxin) -hold digoxin for HR < 90 (infants)and < 60 (adolescents) -dopamine, dobutamine, epinephrine 3. Vasodilators- decrease after load -nitroglycerin, captopril (Capoten), nitroprusside (Nipride)

2-hr PPBS